ID Content Options
12019311

The inverse of the matrix \(\begin{bmatrix} 2 & 5 & 0 \\ 0 & 1 & 1 \\ -1 & 0 &3 \end{bmatrix} \)


Please choose your answer from the right side options

\(\begin{bmatrix} 3 & -15 &5 \\ -1 &6 &-2 \\ 1 &-5 &2 \end{bmatrix} \)

\(\begin{bmatrix} 3 & -1 &1 \\ -15 &6 &-5 \\ 5 &-2 &2 \end{bmatrix} \)

\(\begin{bmatrix} 3 & -15 &5 \\ -1 &6 &-2 \\ 1 &-5 &-2 \end{bmatrix} \)

\(\begin{bmatrix} 3 & -5 &5 \\ -1 &-6 &-2 \\ 1 &-5 &2 \end{bmatrix}\)

12019312

If P and Q are symmetric matrices of the same order then PQ-QP is 


Please choose your answer from the right side options

Zero matrix 

Identity matrix 

Skew symmetric matrix 

Symmetric matrix

12019313

If \(3A+4B'=\begin{bmatrix} 7 &-10 &17 \\ 0 & 6 &31 \end{bmatrix} \) and \(2B-3A'=\begin{bmatrix} -1 & 18\\ 4& 0\\ -5&-7 \end{bmatrix} \) then B =


Please choose your answer from the right side options

(A) \(\begin{bmatrix} -1 & -18\\ 4& -16\\ -5&-7 \end{bmatrix} \) 

\(\begin{bmatrix} 1 & 3\\ -1& 1\\ 2&4 \end{bmatrix} \) 

\(\begin{bmatrix} 1 & 3\\ -1& 1\\ 2&-4 \end{bmatrix} \) 

\(\begin{bmatrix} 1 & -3\\ -1& 1\\ 2&4 \end{bmatrix}\)

12019314

If \(A=\begin{bmatrix} 1 &3 \\ 4& 2 \end{bmatrix},B=\begin{bmatrix} 2 &-1 \\ 1 & 2 \end{bmatrix} \) ,  Then \(\left | ABB \,' \right |=\)


Please choose your answer from the right side options

 100 

50 

250 

-250

12019315

If the value of a third order determinant is 16, then the value of the determinant formed by replacing each of its elements by its cofactor is 


Please choose your answer from the right side options

256 

96 

16 

48

12019316

\(\large\int x^3\,sin\,3xdx= \)


Please choose your answer from the right side options

\(\Large-\frac{x^3.cos\,3x}{3}+\frac{x^2.sin\,3x}{3}+\frac{2xcos\,3x}{9}-\frac{2sin\,3x}{27}+C \)

\(\Large-\frac{x^3.cos\,3x}{3}-\frac{x^2.sin\,3x}{3}+\frac{2xcos\,3x}{9}-\frac{2sin\,3x}{27}+C \)

\(\Large-\frac{x^3.cos\,3x}{3}+\frac{x^2.sin\,3x}{3}-\frac{2xcos\,3x}{9}-\frac{2sin\,3x}{27}+C \)

\(\Large\frac{x^3.cos\,3x}{3}+\frac{x^2.sin\,3x}{3}-\frac{2xcos\,3x}{9}-\frac{2sin\,3x}{27}+C\)

12019317

The area of the region above X-axis included between the parabola \(y^2 = x\) and the circle \( x^2 + y^2 = 2x\) in square units is


Please choose your answer from the right side options

\(\Large\frac{2}{3}-\frac{\pi}{4} \)

\(\Large\frac{\pi}{4}-\frac{3}{2} \)

\(\Large\frac{\pi}{4}-\frac{2}{3} \)

\(\Large\frac{3}{2}-\frac{\pi}{4}\)

12019318

The area of the region bounded by \(Y-axis, y = cos x\) and \(y = sin x\)  , \(\large0\leq x\leq \frac{\pi}{2}\) is 


Please choose your answer from the right side options

\(\sqrt2 +1\,\, Sq. units\) 

\(\sqrt2 -1\,\, Sq. units\) 

\(2-\sqrt2 \,\, Sq. units\) 

\(\sqrt2 \,\, Sq. units\)

12019319

The integrating factor of the differential equation \((2x + 3y^2) \,dy = y \,dx \,\,(y > 0)\) is


Please choose your answer from the right side options

\( \Large{\frac{1}{x}}\)

\(\Large\frac{1}{e^y}\)

\(\Large\frac{1}{y^2}\)

\(-\Large\frac{1}{y^2}\)

12020311

If \(2^x+2^y=2^{x+y}\) , then  \(\Large\frac{dy}{dx}\) is


Please choose your answer from the right side options

\(2^{y-x}\) 

 \(-2^{y-x}\) 

\(2^{x-y}\)

\(\Large\frac{2^y-1}{2^x-1}\)

12020312

If \(f(x)=sin^{-1}\left ({ \Large\frac{2x}{1+x^2}} \right ) \) , then \(f'(\sqrt{3}) \) is


Please choose your answer from the right side options

\(-\Large\frac{1}{2} \)

\(\Large\frac{1}{2} \)

\(\Large\frac{1}{\sqrt{3}} \)

\(-\Large\frac{1}{\sqrt{3}}\)

12020313

The right hand and left hand limit of the function are respectively

\(f{(x)}=\left\{\begin{matrix} \Large\frac{e^{\frac{1}{x}}-1}{e^{\frac{1}{x}}+1} &,\,if\,x\neq 0 \\ 0, & if\,x= 0 \end{matrix}\right.\) 


Please choose your answer from the right side options

1 and 1 

1 and -1 

-1 and -1 

-1 and 1

12020314

If \(y=2x^{n+1}+{\Large\frac{3}{x^n}} \) , then \(x^2{\Large\frac{d^2y}{dx^2}} \) is


Please choose your answer from the right side options

6n(n+1)y 

n(n+1)y 

\(x{\Large\frac{dy}{dx}}+y\) 

y

12020315

If the curves \(2x=y^2\) and 2xy=K intersect perpendicularly, then the value of \(K^2\) is 


Please choose your answer from the right side options

\(2\sqrt2\) 

8

12020316

If \(\left ( xe \right )^y=e^x \) , then is \(\Large\frac{dy}{dx} \) is =


Please choose your answer from the right side options

\(\Large\frac{log\,x}{(1+log\,x)^2} \)

\(\Large\frac{1}{(1+log\,x)^2} \)

\(\Large\frac{log\,x}{(1+log\,x)} \)

\(\Large\frac{e^x}{x(y-1)}\)

12020317

If the side of a cube is increased by 5%, then the surface area of a cube is increased by 


Please choose your answer from the right side options

10% 

60% 

6% 

20%

12020318

The value of \(\int {\large\frac{1+x^4}{1+x^6}}dx \) is


Please choose your answer from the right side options

\(tan^{-1}\,x+tan^{-1}\,x^3+C \)

\(tan^{-1}\,x+{\Large\frac{1}{3}}tan^{-1}\,x^3+C \)

\(tan^{-1}\,x-{\Large\frac{1}{3}}tan^{-1}\,x^3+C \)

\(tan^{-1}\,x+{\Large\frac{1}{3}}tan^{-1}\,x^2+C\)

12020319

The maximum value of \(\Large\frac{log_ex}{x}\) , if x > 0 is


Please choose your answer from the right side options

  e 

  1 

\(\Large\frac{1}{e}\)

\(-\Large\frac{1}{e}\)

102018311

The value of \(\frac{2(cos\,75^{\circ}+isin\,75^{\circ})}{0.2(cos\,30^{\circ}+isin\,30^{\circ})}\) is


Please choose your answer from the right side options

\(\frac{5}{\sqrt{2}}(1+i)\)

\(\frac{10}{\sqrt{2}}(1+i)\)

\(\frac{10}{\sqrt{2}}(1-i)\)

\(\frac{5}{\sqrt{2}}(1-i)\)

\(\frac{1}{\sqrt{2}}(1+i)\)

102018312

If the conjugate of a complex number \(z\) is \(\frac{1}{i-1}\) , then \(z\) is


Please choose your answer from the right side options

\(\frac{1}{i-1}\)

\(\frac{1}{i+1}\)

\(\frac{-1}{i-1}\)

\(\frac{-1}{i+1}\)

\(\frac{1}{i}\)

102018313

The value of \(\left ( i^{18}+\left ( \frac{1}{i} \right )^{25}\right )^{3}\) is equal to


Please choose your answer from the right side options

\(\frac{1+i}{2}\)

\(2+2i\)

\(\frac{1-i}{2}\)

\(\sqrt2-\sqrt2 i\)

\(2-2i\)

102018314

The modulus of \(\frac{1+i}{1-i}-\frac{1-i}{1+i}\) is


Please choose your answer from the right side options

\(2\)

\(\sqrt2\)

\(4\)

\(8\)

\(10\)

102018315

If \(z=e^\frac{i4\pi }{3}\) , then \((z^{192}+z^{194})^3 \) is equal to


Please choose your answer from the right side options

\(-2\)

\(-1\)

\(-i\)

\(-2i\)

\(0\)

102018316

If \(a\) and \(b\) are real numbers and \((a+ib)^{11}=1+3i\) , then \((b+ia)^{11}\) is equal to


Please choose your answer from the right side options

\(i+3\)

\(1+3i\)

\(1-3i\)

\(0\)

\(-i-3\)

102018317

If \(\alpha \neq \beta ,\alpha ^2=5\alpha -3,\beta ^2=5\beta -3\) , then the equation having \(\frac{\alpha}{\beta}\) and \(\frac{\beta}{\alpha}\) as its roots is


Please choose your answer from the right side options

\(3x^2-19x-3=0\)

\(3x^2+19x-3=0\)

\(x^2+19x+3=0\)

\(3x^2-19x-19=0\)

\(3x^2-19x+3=0\)

102018318

The focus of the parabola \(y^2-4y-x+3=0 \) is


Please choose your answer from the right side options

\(\left ( \frac{3}{4},2 \right )\)

\(\left ( \frac{3}{4},-2 \right )\)

\(\left ( 2, \frac{3}{4} \right )\)

\(\left ( \frac{-3}{4},2 \right )\)

\(\left ( 2,\frac{-3}{4} \right )\)

102018319

If \(f:R\rightarrow (0,\infty )\) is an increasing function and if  \(lim_{x\rightarrow 2018}\frac{f(3x)}{f(x)}=1\) , then \(lim_{x\rightarrow 2018}\frac{f(2x)}{f(x)}\) is equal to


Please choose your answer from the right side options

\(\frac{2}{3}\)

\(\frac{3}{2}\)

\(2\)

\(3\)

\(1\)

120193110

The equation of the curve passing through the point (1, 1) such that the slope of the tangent at any point (x, y) is equal to the product of its co-ordinates is


Please choose your answer from the right side options

\(2\,log\,y=x^2-1 \)

\(2\,log\,x=y^2-1 \)

\(2\,log\,x=y^2+1 \)

\(2\,log\,y=x^2+1\)

120193111

Foot of the perpendicular drawn from the point (1, 3, 4) to the plane \(2x - y + z + 3 = 0\) is  


Please choose your answer from the right side options

(1, 2, - 3) 

(-1, 4, 3) 

\( (-3, 5, 2) \)

\((0, -4, -7)\)

120193112

Acute angel between the line \(\Large\frac{x-5}{2}=\frac{y+4}{-1}=\frac{z+4}{1} \) and the plane \(3x - 4y - z + 5 = 0 \) is


Please choose your answer from the right side options

\(\Large\cos^{-1}\left ( \frac{5}{2\sqrt{13}} \right ) \)

\(\Large\cos^{-1}\left ( \frac{9}{\sqrt{364}} \right ) \)

\(\Large\sin^{-1}\left ( \frac{5}{2\sqrt{13}} \right ) \)

\(\Large\sin^{-1}\left ( \frac{9}{\sqrt{364}} \right )\)

Non of the Above

120193113

The distance of the point (1, 2, 1) from the line \(\Large\frac{x-1}{2}=\frac{y-2}{1}=\frac{z-3}{2} \) is


Please choose your answer from the right side options

 \(\Large\frac{\sqrt{5}}{3} \)

 \(\Large\frac{2\sqrt{3}}{5} \) 

\(\Large\frac{{20}}{3} \) 

\(\Large\frac{2\sqrt{5}}{3}\)

120193114

XY – plane divides the line joining the points A (2, 3, - 5) and B (-1, -2, -3) in the ratio 


Please choose your answer from the right side options

5 : 3 internally 

2 : 1 internally 

5 : 3 externally 

3 : 2 externally

120193115

The shaded region in the figure is the solution set of the inequations.


Please choose your answer from the right side options

\(\large4x + 5y ≤ 20, 3x + 10y ≤ 30, x ≤ 6, x, y ≥ 0 \)

\(\large4x + 5y ≥ 20, 3x + 10y ≤ 30, x ≤ 6, x, y ≥ 0 \)

\(\large4x + 5y ≤ 20, 3x + 10y ≤ 30, x ≥ 6, x, y ≥ 0 \)

\(\large4x + 5y ≥ 20, 3x + 10y ≤ 30, x ≥ 6, x, y ≥ 0\)

120193116

The constant term in the expansion of \(\begin{bmatrix} 3x+1 &2x-1 &x+2 \\ 5x-1&3x+2 & x+1\\ 7x-2 & 3x+1 & 4x-1 \end{bmatrix}\) is 


Please choose your answer from the right side options

– 10 

2

120193117

If \([x]\) represents the greatest integer function and \(f (x) = x - [x] – cos x\) then \(\Large f'(\Large\frac{\pi}{2})\)=


Please choose your answer from the right side options

      2 

    0 

does not exist 

    1

120193118

If \(\Large f(x)=\left\{\begin{matrix} \frac{sin\,3x}{e^{2x}-1} &;x\neq 0 \\ k-2 &; x=0 \end{matrix}\right.\) is Continuous at x = 0, then k =


Please choose your answer from the right side options

\(\Large\frac{1}{2}\)

\(\Large\frac{3}{2}\)

\(\Large\frac{2}{3}\)

\(\Large\frac{9}{5}\)

Non of the Above

120193119

If  \(\Large f(x)=sin^{-1}\left [ \frac{2^{x+1}}{1+4^x} \right ]\) , then \(f’ (0) =\)  


Please choose your answer from the right side options

\(\Large \frac{2\,log\,2}{5} \) 

\(\Large2\,log\,2 \) 

\(\Large \frac{4\,log\,2}{5} \) 

\(\Large log\,2 \)

120193120

If \(x=1\,sec^2\,\theta,y=a\,tan^2\theta \)  then \(\Large \frac{d^2y}{dx^2}=\)


Please choose your answer from the right side options

  0

  2a

  4

  1

120193121

If \(α\) and \(β\) are roots of the equation \(\large x^2 = x + 1 = 0\) then \(α^ 2 + β ^ 2\) is 


Please choose your answer from the right side options

\(\Large\frac{-1-i\sqrt{3}}{2} \)

 \(1 \)

 \(-1 \)

\(\Large\frac{-1+i\sqrt{3}}{2}\)

120193122

The number of 4 digit numbers without repetition that can be formed using the digits 1, 2, 3, 4, 5, 6, 7 in which each number has two odd digits and two even digits is


Please choose your answer from the right side options

450 

432 

454 

436

120193123

The number of terms in the expansion of \(\large(x^2+y^2)^{25}-(x^2-y^2)^{25}\) after simplification is 


Please choose your answer from the right side options

  26 

  0 

  50 

  13

120193124

The third term of a G.P. is 9. The product of its first five terms is 


Please choose your answer from the right side options

\(3^{10}\) 

\(3^5\) 

\(3^{12}\) 

\(3^9\)

120193125

A line cuts off equal intercepts on the co-ordinate axes. The angle made by this line with the positive direction of X-axis is 


Please choose your answer from the right side options

120° 

45° 

135° 

90°

120193126

The eccentricity of the ellipse \(9x^2 + 25y^2 = 225\) is 


Please choose your answer from the right side options

\(\Large\frac{3}{4} \)

 \(\Large\frac{4}{5} \)

 \(\Large\frac{9}{16} \) 

\(\Large\frac{3}{5}\)

120193127

\(\Large\sum_{r=1}^{n}(2r-1)=x \)  then  \(\Large\lim_{n\rightarrow \infty }\left [\frac{1^3}{x^2}+\frac{2^3}{x^2}+\frac{3^3}{x^2}+......+\frac{n^3}{x^2} \right ]=\)


Please choose your answer from the right side options

  1

  \(\Large\frac{1}{2}\)

  4

\(\Large\frac{1}{4}\)

120193128

The negative of the statement “All continuous functions are differentiable.” 


Please choose your answer from the right side options

Some continuous functions are not differentiable 

All continuous functions are not differentiable. 

All differentiable functions are continuous. 

Some continuous functions are differentiable

120193129

Mean and standard deviation of 100 items are 50 and 4 respectively. The sum of all squares of the items is 


Please choose your answer from the right side options

266000 

251600 

261600 

256100

120193130

Two letters are chosen from the letters of the word ‘EQUATIONS’. The probability that one is vowel and the other is consonant is


Please choose your answer from the right side options

\(\Large\frac{3}{9}\)

\(\Large\frac{8}{9}\)

\(\Large\frac{5}{9}\)

\(\Large\frac{4}{9}\)

120193131

\(f: R → R\) and \(g : [0, ∞) → R\) is defined by \(f(x) = x^2\) and \(g(x) =\sqrt{x}\) . Which of the following is not true?


Please choose your answer from the right side options

 fog (2) = 2 

gof (4) = 4 

gof (-2) = 2 

fog (-4) = 4

120193132

If \(A = \{x| x ∈ N, x ≤ 5\}, B =\{ x | x ∈ Z, x^2 – 5x + 6 = 0\}\), then the number of onto functions from A to B is 


Please choose your answer from the right side options

30 

32 

23

120193133

On the set of positive rationals, a binary operation \(\large *\) is defined by \(\Large a * b =\frac{2ab}{5}\). If \(\large2 * x = 3^{-1}\) then \(x=\)


Please choose your answer from the right side options

\(\Large\frac{2}{5} \)

\(\Large\frac{1}{6} \)

\(\Large\frac{125}{48} \)

\( \Large\frac{5}{12}\)

120193134

\(\Large cos\left [ 2\,sin^{-1}\frac{3}{4}+ cos^{-1}\frac{3}{4}\right ]= \)


Please choose your answer from the right side options

\(\Large\frac{3}{5} \)

\(-\Large\frac{3}{4} \)

does not exist

\(\Large\frac{3}{4}\)

120193135

If \(\large a+\Large\frac{\pi}{2}<2\,tan^{-1}\,x+3\,cot^{-1}\,x<b\) then ‘a’ and ‘b’ are respectively.


Please choose your answer from the right side options

0 and 2π 

0 and π 

\(-\Large \frac{\pi}{2}\) and \(\Large \frac{\pi}{2}\)

\(\Large \frac{\pi}{2}\)and \(\Large\2pi\)

120193136

If \(|3x – 5| ≤ 2\) then


Please choose your answer from the right side options

\(\Large1\leq x\leq \frac{9}{3} \)

\(\Large-1\leq x\leq \frac{7}{3}\)

\(\Large-1\leq x\leq \frac{9}{3} \)

\(\Large1\leq x\leq \frac{7}{3}\)

120193137

A random variable ‘X’ has the following probability distribution:

X  1 2 3 4 5 6 7
P(X) k-1 3k k 3k \(3k^2\) \(k^2\) \(k^2+k\)

Then the value of k is


Please choose your answer from the right side options

\(\Large \frac{2}{7}\)

\(\Large\frac{1}{5}\)

\(\Large\frac{1}{10}\)

\(-2\)

120193138

If A and B are two events of a sample space S such that P(A) = 2.0, P(B) = 0.6 and P(A|B) = 0.5 then P(A’|B)=


Please choose your answer from the right side options

\(\Large\frac{1}{2}\)

\(\Large\frac{3}{10}\)

\(\Large\frac{1}{3}\)

\(\Large\frac{2}{3}\)

120193139

If ‘X’ has a binomial distribution with parameters n = 6, p and P(X = 2) = 12, P(X = 3) = 5 then P =


Please choose your answer from the right side options

\(\Large\frac{1}{2}\)

\(\Large\frac{5}{12}\)

\(\Large\frac{5}{16}\)

\(\Large\frac{16}{21}\)

Non of the Above

120193140

A man speaks truth 2 out of 3 times. He picks one of the natural numbers in the set S = {1, 2, 3, 4, 5, 6, 7} and reports that it is even. The probability that it is actually even is


Please choose your answer from the right side options

\(\Large\frac{1}{10}\)

\(\Large\frac{2}{5}\)

\(\Large\frac{3}{5}\)

\(\Large\frac{1}{5}\)

120193141

The order of the differential equation \(\large y=C_1e^{C_2+x}+C_3e^{C_4+x}\) is


Please choose your answer from the right side options

  3

  1

  4

  2

120193142

If \(\large\left | \vec{a} \right |=16,\left | \vec{b} \right |=4 \) , then, \(\large \sqrt{\left | \vec{a}\times \vec{b} \right |^2+ \left | \vec{a}. \vec{b} \right |^2}=\)


Please choose your answer from the right side options

  16

  4

  64

  8

120193143

If the angle between \(\vec{a} \)and \(\vec{b} \)  is \(\Large\frac{2\pi}{3} \) and the projection of \(\vec{a} \) in the direction of \(\vec{b} \) is -2, the \(\left | \vec{a} \right |\)=


Please choose your answer from the right side options

  2

  4

  1

  3

120193144

A unit vector perpendicular to the plane containing the vectors  \(\hat{i}+2\hat{j}+\hat{k} \)  and  \(-2\hat{i}+\hat{j}+3\hat{k} \)  is


Please choose your answer from the right side options

\(\Large\frac{-\hat{i}+\hat{j}-\hat{k}}{\sqrt{3}} \)

\(\Large\frac{\hat{i}+\hat{j}+\hat{k}}{\sqrt{3}} \)

\(\Large\frac{-\hat{i}-\hat{j}-\hat{k}}{\sqrt{3}} \)

\(\Large\frac{-\hat{i}+\hat{j}-\hat{k}}{\sqrt{3}}\)

120193145

\(\large\left [ \vec{a}+2\vec{b}-\vec{c} ,\vec{a}-\vec{b},\vec{a}-\vec{b}-\vec{c}\right ]= \)


Please choose your answer from the right side options

\(\large2\left [ \vec{a},\vec{b},\vec{c} \right ] \)

  0

\(\large3\left [ \vec{a},\vec{b},\vec{c} \right ] \)

\(\large\left [ \vec{a},\vec{b},\vec{c} \right ]\)

120193146

\(\Large\sqrt[3]{y}\sqrt{x}\,\,\,\sqrt[6]{(x+y)^5} \), then \(\Large\frac{dy}{dx}=\) 


Please choose your answer from the right side options

\(\large x-y\)

\(\Large\frac{x}{y}\)

\(\Large\frac{y}{x}\)

\(\Large x+y\)

120193147

Rolle’s theorem is not applicable in which one of the following cases?


Please choose your answer from the right side options

\( f (x) = |x| \) in \( [-2, 2]\)

\(f(x) = x^2 – 4x + 5\) in [1, 3] 

\(f (x) = [x]\) in [2.5, 2.7] 

\(f(x) = x^2 – x\) in [0,1]

120193148

The interval in which the function \(f (x) = x^3 – 6x^2 + 9x + 10\) is increasing in


Please choose your answer from the right side options

 [1, 3] 

\((-∞, 1) ∪ (3, ∞)\) 

\((-∞, -1] ∪ [3, ∞)\)  

\((-∞, 1] ∪ [3, ∞)\)

120193149

The sides of an equilateral triangle are increasing at the rate of 4 cm/sec. The rate at which its area is increasing, when the side is 14 cm. 


Please choose your answer from the right side options

\(42\, cm^2 /sec\) 

\(10\sqrt{3}\, cm^2 /sec\) 

\(14 \,cm^2 /sec\) 

\(14\sqrt{3}\, cm^2 /sec\)

Non of the Above

120193150

The value of \(\sqrt{24.99}\) is


Please choose your answer from the right side options

5.001 

4.999 

4.897 

4.899

Non of the Above

120193151

\(\Large\int_{-3}^{3} cot^{-1}x\,dx =\)


Please choose your answer from the right side options

\(\large6 \pi\)

\(\large3\pi\)

  3

  0

120193152

\(\Large\int \frac{1}{\sqrt{x}+x\sqrt{x}}\,dx= \)


Please choose your answer from the right side options

\( tan^{-1}\sqrt{x} + C \)

\(2\, log ( \sqrt{x }+ 1)+ C \)

\(2 tan^{-1}\sqrt{x} + C \)

\({\Large\frac{1}{2}}tan^{-1}\sqrt{x}+C \)

120193153

\({\Large\int \frac{2x-1}{(x-1)(x^2)(x-3)}}dx=A\,log\,\left | x-1 \right |+B\,log\,\left | x+2 \right |+C\,log\,\left | x-3 \right |+K\)

Then A, B, C are respectively.


Please choose your answer from the right side options

\(\Large\frac{1}{6},\frac{-1}{3},\frac{1}{3} \)

\(\Large\frac{-1}{6},\frac{1}{3},\frac{1}{3} \)

\(\Large\frac{-1}{6},\frac{-1}{3},\frac{1}{2} \)

\(\Large\frac{1}{6},\frac{1}{3},\frac{1}{5} \)

120193154

\(\Large\int_{0}^{2}\left [ x^2 \right ]dx= \)


Please choose your answer from the right side options

\(\large5-\sqrt{2}+\sqrt{3} \)

\(\large5-\sqrt{2}-\sqrt{3} \)

\(-\large5-\sqrt{2}-\sqrt{3}\)

\( \large5+\sqrt{2}-\sqrt{3}\)

120193155

\(\Large\int_{0}^{1}\sqrt{\frac{1+x}{1-x}}dx=\)


Please choose your answer from the right side options

\(\Large\frac{\pi}{2}\)

\({\Large\frac{\pi}{2}}-1\)

\(\Large\frac{1}{2}\)

\({\Large\frac{\pi}{2}}+1\)

120193156

If U is the universal set with 100 element; A and B are two set such that n(A) = 50, n (B) = 60, n (A∩B) = 20 then n (A’∩B’) = 


Please choose your answer from the right side options

  90 

  40 

  10 

 20

120193157

The domain of the function \(f : R → R\) defined by \(f (x)=\sqrt{x^2-7x+12}\) is 


Please choose your answer from the right side options

\((-∞, 3] ∩ [4, ∞)\) 

\((-∞, 3] ∪ [4, ∞)\) 

(3, 4) 

\((-∞, 3] ∪ (4, ∞)\)

120193158

If \(cos \,x = |sin \,x|\) then, the general solution is


Please choose your answer from the right side options

\(\large x=n\pi+(-1)^n\,\,{\Large\frac{\pi}{4}},n\epsilon Z \)

\(\large x=n\pi\pm \,\,{\Large\frac{\pi}{4}},n\epsilon Z \)

\(\large x= (2n=1)\pi\pm \,\,{\Large\frac{\pi}{4}},n\epsilon Z \)

\(\large x=2n\pi\pm \,\,{\Large\frac{\pi}{4}},n\epsilon Z\)

120193159

\(\large \sqrt{3}\,\,cosec\,20^{\circ}-sec\,20^{\circ}=\)


Please choose your answer from the right side options

  4

  2

  1

  3

120193160

If \(P (n) : 2^n < n!\) , Then the smallest positive integer for which P (n) is true, is


Please choose your answer from the right side options

  4

  2

  5

  3

120203110

The value of \(\int e^{sin\,x}\,sin\,2x\,dx \) is


Please choose your answer from the right side options

\(2e^{sin\,x}(sin\,x-1)+C \)

\(2e^{sin\,x}(sin\,x+1)+C \)

\(2e^{sin\,x}(cos\,x+1)+C \)

\(2e^{sin\,x}(cos\,x-1)+C\)

120203111

The value of \(\int_{-\frac{1}{2}}^{\frac{1}{2}}cos^{-1}\,x\,dx \) is 


Please choose your answer from the right side options

π

\(\Large\frac{\pi}{2} \) 

1  

\(\Large\frac{\pi^2}{2}\)

120203112

If \(\int{ \Large\frac{3x+1}{(x-1)(x-2)(x-3)}}\,dx=\,A\,log\,\left | x-1 \right |B\,log\,\left | x-2 \right |+C\,log\,\left | x-3 \right |\) , then the values of A, B and C are respectively 


Please choose your answer from the right side options

5, -7, -5 

2, -7, -5 

5, -7, 5 

2, -7, 5

120203113

The value of \({\Large\int_{0}^{1}}{\Large\frac{log(1+x)}{1+x^2}}dx \) is


Please choose your answer from the right side options

\({\Large\frac{\pi}{2}}log\,2 \)

\({\Large\frac{\pi}{4}}log\,2 \)

\(\Large\frac{1}{2} \)

\({\Large\frac{\pi}{8}}log\,2\)

120203114

The area of the region bounded by the curve \(y^2 =8x\) and the line \(y=2x\) is 


Please choose your answer from the right side options

\(\Large\frac{16 }{ 3}\) sq .units 

\(\Large\frac{4}{ 3}\) sq .units 

\(\Large\frac{3 }{ 4}\) sq. units 

\(\Large\frac{8 }{ 3}\) sq .units

120203115

The value of \({\Large\int_{-\frac{\pi }{2}}^{\frac{\pi }{2}}\frac{cos\,x}{1+e^x}}dx\) is 


Please choose your answer from the right side options

-2

120203116

The order of the differential equation obtained by eliminating arbitrary constants in the family of curves  \(c_1y=(c_2+c_3)e^{x+c_4}\) is


Please choose your answer from the right side options

1

3

4

120203117

The general solution of the differential equation \(\large x^2dy-2xydx=x^4\,cos\,xdx \) is


Please choose your answer from the right side options

\(\large y=x^2\,sin\,x+cx^2 \)

\(\large y=x^2\,sin\,x+c \)

\(\large y=\,sin\,x+cx^2 \)

\(\large y=\,cos\,x+cx^2\)

120203118

The area of the region bounded by the line \(y=2x+1\), x−axis and the ordinates x=−1 and x=1 is 


Please choose your answer from the right side options

\(\Large\frac{9}{4}\) 

\(\Large\frac{5}{2}\) 

5

120203119

The two vectors \(\hat{i}+\hat{j}+\hat{k} \) and \(\hat{i}+3\hat{j}+5\hat{k} \) represent the two sides \(\overline{AB} \)  and \(\overline{AC} \) respectively of a \(ΔABC\). The length of the median through A is 


Please choose your answer from the right side options

\(\Large\frac{\sqrt{14}}{2} \) 

14 

\(\sqrt{14}\)

120203120

If \(\vec{a} \) and \(\vec{b} \) are unit vectors and \(θ\) is the angle between \(\vec{a} \) and \(\vec{b} \) then \(sin\,\Large​​\frac{\theta}{2}\)


Please choose your answer from the right side options

\(\left | \vec{a} +\vec{b} \right | \)

\(\Large\frac{\left | \vec{a} +\vec{b} \right | }{2} \)

\(\Large\frac{\left | \vec{a} -\vec{b} \right | }{2} \)

\(\left | \vec{a} -\vec{b} \right |\)

120203121

The curve passing through the point (1, 2) given that the slope of the tangent at any point (x,y) is \(\Large\frac{3x}{y}\) represents 


Please choose your answer from the right side options

Circle 

Parabola 

Ellipse 

Hyperbola

120203122

If \(\large\left | \vec{a}+\vec{b} \right |^2+\left | \vec{a}\cdot \vec{b} \right |^2=144\left | \vec{a} \right |=6 \) then \(\large\left | \vec{b} \right |\) is equal to 


Please choose your answer from the right side options

4

120203123

The point (1, -3, 4) lies in the octant 


Please choose your answer from the right side options

Second 

Third 

Fourth 

Eighth

120203124

If the vectors \(2\hat{i}-3\hat{j}+4\hat{k} \,,2\hat{i}+\hat{j}-\hat{k} \)  and \(\lambda \hat{i}-\hat{j}+2\hat{k}\) are coplanar, then the value of \(\lambda\) is 


Please choose your answer from the right side options

-5 

-6 

5

120203125

The distance of the point (1, 2, -4) from the line \(\Large\frac{x-3}{2}=\frac{y-3}{3}=\frac{z+5}{6} \) is


Please choose your answer from the right side options

\(\Large\frac{293}{7} \)

\(\Large\frac{\sqrt{293}}{7} \)

\(\Large\frac{293}{49} \)

\(\Large\frac{\sqrt{293}}{49}\)

120203126

The sine of the angle between the straight line \(\Large\frac{x-2}{3}=\frac{3-y}{-4}=\frac{z-4}{5} \) and the plane \(2x−2y+z=5\)  is


Please choose your answer from the right side options

\(\Large\frac{3}{\sqrt{30}} \)

\(\Large\frac{{3}}{50} \)

\(\Large\frac{4}{5\sqrt2} \) 

\(\Large\frac{\sqrt{2}}{10}\)

120203127

If a line makes an angle of \(\Large\frac{\pi}{3}\) with each of x and and y−axis, then the acute angle made by z−axis is


Please choose your answer from the right side options

\(\Large \frac{\pi}{4} \)

\(\Large\frac{\pi}{6} \)

\(\Large\frac{\pi}{3} \)

\(\Large\frac{\pi}{2}\)

120203128

Corner points of the feasible region determined by the system of linear constraints are (0, 3), (1, 1) and (3, 0). Let z = px = qy , where p, q>0.  Condition on p and q so that the minimum of z occurs at (3, 0) and (1, 1) is 


Please choose your answer from the right side options

p=2q 

\(p=\Large\frac{q}{2}\)

p=3q 

p=q

120203129

The feasible region of an LPP is shown in the figure. If \(Z = 11x + 7y\) , then the maximum value of Z occurs at 


Please choose your answer from the right side options

(0,5) 

(3,3) 

(5,0) 

(3,2)

120203130

A die is thrown 10 times, the probability that an odd number will come up at least one time is


Please choose your answer from the right side options

\(\Large\frac{1}{1024} \)

\(\Large\frac{1023}{1024} \)

\(\Large\frac{11}{1024} \)

\(\Large\frac{1013}{1024}\)

120203131

If A and B are two events such that \(P(A)={\Large\frac{1}{3}},P(B)=\Large\frac{1}{2} \) and \(P(A\cap B)=\Large\frac{1}{6} \) , then \(P(A'/B)\)  is


Please choose your answer from the right side options

\(\Large\frac{2}{3} \)

\(\Large\frac{1}{3} \)

\(\Large\frac{1}{2} \)

\(\Large\frac{1}{12}\)

120203132

Events \(E_1\) and \(E_2\) from a partition of the sample space S. A is any event such that \(P(E_1)=P(E_2)={\Large\frac{1}{2}},P(E_2/A)={\Large\frac{1}{2}} \)and \(P(A/E_2)={\Large\frac{2}{3}} \) , then \(P(E_1/A) \) is


Please choose your answer from the right side options

\({\Large\frac{1}{2}} \)

\({\Large\frac{2}{3}} \)

\(1 \)

\({\Large\frac{1}{4}}\)

120203133

The probability of solving a problem by three persons A, B and C independently is \(\Large\frac{1}{2},\Large\frac{1}{4}\) and \(\Large\frac{1}{3}\) respectively. Then the probability of the problem is solved by any two of them is 


Please choose your answer from the right side options

\(\Large\frac{1}{12}\) 

\(\Large\frac{1}{4}\) 

\(\Large\frac{1}{24}\) 

\(\Large\frac{1}{8}\)

120203134

If n(A) = 2 and total number of possible relations from Set A to set B is 1024, then n(B) is 


Please choose your answer from the right side options

512 

20 

10 

5

120203135

The value of \(\large sin^2\,51^{\circ}+sin^2\,39^{\circ}\) is 


Please choose your answer from the right side options

1

0  

\(sin12^{\circ} \) 

\(cos12^{\circ}\)

120203136

If \(tan A + cot A = 2\), then the value of \(tan^4 A + cot ^4A = \) 


Please choose your answer from the right side options

5

120203137

If A = {1,2,3,4,5,6}, then the number of subsets of A which contain at least two elements is 


Please choose your answer from the right side options

64 

63

57 

58

120203138

If z = x+iy, then the equation |z+1| = |z-1| represents 


Please choose your answer from the right side options

a circle 

a parabola 

x-axis 

y-axis

120203139

The value of \(^{16}C_9+^{16}C_{10}-^{16}C_6-^{16}C_7 \)  is 


Please choose your answer from the right side options

0

1

\(^{17}C_{10}\)

\(^{17}C_{3}\)

120203140

The number of terms in the expansion of \((x+y+z)^{10}\) is 


Please choose your answer from the right side options

66 

142 

11 

110

120203141

If \(P(n):2^n<n!\) Then the smallest positive integer for which P(n) is true , is


Please choose your answer from the right side options

2

3

4

5

120203142

The two lines \(lx+my=n\) and \(l'x+m'y=n'\) are perpendicular if 


Please choose your answer from the right side options
 

'>

\(ll'+mm'=0\) 

 

'>

\( lm'+ml'\) 

 

'>

\(lm+l'm'=0\) 

'>

\(lm'+ml'=0\)

120203143

If the parabola \(x^2=4ay\) passes through the point (2, 1), then the length of the latus rectum is 


Please choose your answer from the right side options

1  

8

120203144

If the sum of n terms of an A.P is given by \(S_n=n^2 +n\), then the common difference of the A.P is 


Please choose your answer from the right side options

6

120203145

The negation of the statement "For all real numbers x and y, \(x + y = y + x\) " is 


Please choose your answer from the right side options

For all real numbers x and y, \(x+y≠y+x\) 

For some real numbers x and y, \(x+y = y+x\) 

For some real number x and y, \(x + y ≠ y + x\) 

for some real numbers x and y, \(x-y=y-x\)

120203146

The standard deviation of the data 6, 7, 8, 9, 10 is 


Please choose your answer from the right side options

\(\sqrt2\)

\(\sqrt{10}\) 

10

120203147

\(\lim_{x\rightarrow 0}\left ( \Large\frac{tan\,x}{\sqrt{2x+4}-2} \right )\) is equal to 


Please choose your answer from the right side options

2

3

4

6

120203148

If a relation R on the set {1, 2, 3} be defined by R={(1, 1)}, then R is 


Please choose your answer from the right side options

Reflexive and symmetric 

Reflexive and transitive 

symmetric and transitive 

Only symmetric

120203149

Let \(f:[2,∞)→R\) be the function defined \(f(x)=x^2 −4x+5\) , then the range of f is 


Please choose your answer from the right side options

\((−∞,∞)\) 

\([1,∞) \)

\((1,∞)\) 

\([5,∞)\)

120203150

If A, B, C are three mutually exclusive and exhaustive events of an experiment such that P(A)= 2 P(B) = 3P(C), then P(B) is equal to 


Please choose your answer from the right side options

\(\Large\frac{1}{11} \)

\(\Large\frac{2}{11} \)

\(\Large\frac{3}{11} \)

\(\Large\frac{4}{11}\)

120203151

The domain of the function defined by \(f(x)=cos^{-1}\sqrt{x-1 }\) 


Please choose your answer from the right side options

[1, 2] 

[0, 2] 

[-1, 1] 

[0, 1]

120203152

The value of \(cos\left ( sin^{-1}{\Large\frac{\pi}{3}}+cos^{-1} {\Large\frac{\pi}{3}}\right )\) is 


Please choose your answer from the right side options

-0 

Does not exist

120203153

If \(A=\begin{pmatrix} 0 &0 &1 \\ 0 & 1 & 0\\ 1 &0 &0 \end{pmatrix}\) then \(A^4\) is equal to 


Please choose your answer from the right side options

2A 

4A

120203154

If A = {a,b,c}, then the number of binary operations on A is 


Please choose your answer from the right side options

\(3^6 \)

\(3^3 \)

\(3^9\)

120203155

If \(\begin{pmatrix} 2 &1 \\ 3 & 2 \end{pmatrix}A=\begin{pmatrix} 1 &0 \\ 0&1 \end{pmatrix} \) , then the matrix a is


Please choose your answer from the right side options

\(\begin{pmatrix} 2 &1 \\ 3&2 \end{pmatrix} \)

\(\begin{pmatrix} 2 &-1 \\ -3&2 \end{pmatrix} \)

\(\begin{pmatrix} -2 &1 \\ 3&-2 \end{pmatrix} \)

\(\begin{pmatrix} 2 &-1 \\ 3&2 \end{pmatrix}\)

120203156

If \(f(x)=\begin{vmatrix} x^3-x &a+x & b+x\\ x-a& x^2-x & c+x\\ x-b& x-c &0 \end{vmatrix}\) then 


Please choose your answer from the right side options

f(1)=0 

f(2)=0  

f(0)=0 

f(-1)=0

120203157

If A and B are square matrices of same order and B is a skew symmetric matrix, then \(A’BA\) is 


Please choose your answer from the right side options

Symmetric matrix 

Null matrix 

Diagonal matrix 

Skew symmetric matrix

120203158

If A is a square matrix of order 3 and |A| = 5, then |A adj.A| is 


Please choose your answer from the right side options

125 

25 

625

120203159

If \(f(n)=\left\{\begin{matrix} {\Large\frac{1-cos\,Kx}{x\,sinx}} ,&if\,x\neq 0 \\ {\Large\frac{1}{2}} &, if\,x=0 \end{matrix}\right.\) 


Please choose your answer from the right side options

\(\pm \Large\frac{1}{2}\) 

\(±2\) 

\(±1\)

120203160

If \(a_1a_2a_3......a_9 \) are in A.P. then the value of \(\begin{vmatrix} a_1 & a_2 &a_3 \\ a_4 &a_5 & a_6\\ a_7 & a_8 & a_9 \end{vmatrix} \) is


Please choose your answer from the right side options

\({\Large\frac{9}{2}}(a_1+a_0) \)

\((a_1+a_9) \)

\(log_e(log_ee)\) 

1

122019311

\(\Large f(x) = \frac{x}{e^x -1} + \frac{x}{2} + 2 \ cos^3 \frac{x}{2}\) on R - {0} is


Please choose your answer from the right side options

One one function

Bijection

Algebraic function

Even function

122019312

Consider the following lists

            List I                                             List II

A) \(\Large f(x) = \frac{|x+2|}{x+2}, x \neq -2\)          I) \(\Large [\frac{1}{3}, 1]\)

B) \(\large g(x) = |[x]|, x \ \epsilon \) ℝ                          II) Z

C) \(\large h(x) = |x-[x]|, x \ \epsilon\) â„                 III) W

D) \(\Large f(x) = \frac{1}{2 - sin \ 3x} , x \epsilon\) ℝ             IV) [0,1)

                                                               V) {-1,1}


Please choose your answer from the right side options

A  B   C   D

V  III   II   I

A   B   C   D
III   II   IV  I

A   B   C   D

V  III   IV   I 

A  B   C  D

I   II   III  IV

122019313

Assertion (A): (1) + (1 + 2 + 4) + (4 + 6 + 9) + ( 9 + 12 + 16) + ... + (81 + 90 + 100) = 1000

Reason (R): \(\large \overset{n} {\underset{r=1} \Sigma } (r^3 - (r - 1)^3 ) = n^3\) for any natural number n .


Please choose your answer from the right side options

Both (A) and (R) are true and (R) is the correct explanation of (A).

Both (A) and (R) are true and (R) is not the correct explanation of (A).

(A) is true but (R) is false.

(A) is false but (R) is true.

122019314

If A = \(\large \begin{bmatrix} 1 & 0 \\ 0 & -1 \end{bmatrix} \) , P = \(\large \begin{bmatrix} 1 & 1 \\ 0 & 1 \end{bmatrix} \) and \(\large X= APA^T\) , then \(\large A^T X^{50} A =\) 


Please choose your answer from the right side options

\(\large \begin{bmatrix} 0 & 1 \\ 1 & 0 \end{bmatrix} \)

\(\large \begin{bmatrix} 2 & 1 \\ 0 & -1 \end{bmatrix} \)

\(\large \begin{bmatrix} 25 & 1 \\ 1 & -25 \end{bmatrix} \)

\(\large \begin{bmatrix} 1 & 50 \\ 0 & 1 \end{bmatrix} \)

122019315

If [x] is the greatest integer less than or equal to x and |x| is the modulus of x, then the system of three equations 

2x +3|y|+ 5[z] = 0, x + |y| - 2[z] = 4, x+ |y| + [z] =1 has,


Please choose your answer from the right side options

a unique solution

finitely many solutions

infinitely many solutions

no solutions

122019316

Investigate the λ and μ for the system x + 2y + 3z =6, x + 3y + 5z = 9, 2x + 5y + λz = μ and match the following values in List I with the items in List II

          List I                             List II

A) \(\large \lambda = 8, \mu \neq 15\)       I) Infinitely many solutions

B) \(\large \lambda \neq 8, \mu \ \epsilon \)  ℝ          II) No solutions

C) \(\large \lambda = 8, \mu = 15\)       III) Unique solutions

The correct match is


Please choose your answer from the right side options

A   B   C

I    II   III

A   B  C

II   III   I

A   B   C

III   I    II

A   B   C

III   II   I

122019317

If \(\large z = x \ + \ iy, \ x,y \ \epsilon R, \ (x,y) \neq (0, -4)\) and \(\Large Arg(\frac{2z-3}{z+4i}) = \frac{\pi}{4}\) , then the locus of z is


Please choose your answer from the right side options

\(\large 2x^2 + 2y^2 + 5x +5y - 12=0\)

\(\large 2x^2 - 3xy + y^2 + 5x + y - 12=0\)

\(\large 2x^2 + 3xy + y^2 + 5x + y + 12=0\)

\(\large 2x^2 + 2y^2- 11x + 7y - 12 = 0\)

122019318

If \(\large z = x+iy, \ x,y \ \epsilon R\) and the imaginary part of \(\Large \frac{\bar z -1 }{\bar z - i}\) is 1 then locus of z is


Please choose your answer from the right side options

x + y + 1 =0

\(\large x + y+1=0, (x,y) \neq (0,-1)\)

\(\large x^2 + y^2 - x + 3y +2 = 0\)

\(\large x^2 + y^2 - x + 3y +2 = 0, (x,y) \neq (0,-1)\)

122019319

If ω represents a complex cube root of unity, then \(\Large (1+\frac{1}{\omega})(1+\frac{1}{\omega^2}) + (2+\frac{1}{\omega})(2+\frac{1}{\omega^2})+ ... + (n+\frac{1}{\omega})(n+\frac{1}{\omega^2}) =\)


Please choose your answer from the right side options

\(\Large \frac{n(n^2 +1)}{3}\)

\(\Large \frac{n(n^2 +2)}{3}\)

\(\Large \frac{n(n^2 -2)}{3}\)

\(\Large \frac{n^2(n-1)}{6}\)

152017311

If  \(tan\,20^{\circ}=\lambda \) ,  then  \(\frac{tan\,160^{\circ}-tan\,110^{\circ}}{1+(tan\,160^{\circ})(tan\,110^{\circ})}= \)


Please choose your answer from the right side options

\(\Large\frac{1+\lambda ^2}{2\lambda } \)

\(\Large \frac{1+\lambda ^2}{\lambda } \)

\(\Large\frac{1-\lambda ^2}{\lambda } \)

\(\Large\frac{1-\lambda ^2}{2\lambda }\)

152017312

Consider the circle \(x^2+y^2-6x+4y=12 \). The equation of a tangent to this circle that is parallel to the line \(4x+3y+5=0 \) is 


Please choose your answer from the right side options

\(4x +3y+ 10= 0 \) 

\(4x+3y-9=0 \) 

\(4x+3y+9=0 \) 

\(4x+3y-31=0\)

152017313

The mean deviation from the mean 10 of the data 6,7,10,12,13, \( \alpha\), 12,16 is 


Please choose your answer from the right side options

3.5 

3.25 

3.75

152017314

Match the following.

List-I  List-II
\(\int_{-1}^{1}x\left | x \right |dx \) \(\frac{\pi}{2} \)
\(\int_{0}^{\frac{\pi}{2}}\left ( 1+log\left ( \frac{4+3sinx}{4+3cosx} \right ) \right )dx \) \( \int_{0}^{\frac{a}{2}}f(x)dx \)
\(\int_{0}^{a}f(x)dx \) \( \int_{0}^{a}[f(x)+f(-x)]dx \)
\(\int_{-a}^{a}f(x)dx \)  \( 0 \)
  \( \int_{0}^{a}f(a-x)dx \)

 


Please choose your answer from the right side options

I II III IV

d a e c

I II III IV

d a c b 

I II III IV

d c a e 

I II III IV

a d b d

152017315

If f is differentiable,\(f(x+y)=f(x)f(y) \) for all \(x,y\,\epsilon R,f(3)=3,f'(0)=11,\) then\(f'(3)=\)


Please choose your answer from the right side options

\(\frac{3}{11}\) 

\(\frac{11}{3}\)

8

33

152017316

\(\Large \int_{0}^{\pi }\frac{xdx}{4cos^2x+9sin^2x}= \) 


Please choose your answer from the right side options

\(\frac{\pi^2}{12}\) 

\(\frac{\pi^2}{4}\)

 \(\frac{\pi^2}{6}\) 

\(\frac{\pi^2}{3}\)

152017317

The probability distribution of a random variable X is given below.

x =k 0 1 2 3 4
P(X=k) 0.1 0,4 0.3 0.2 0

The variance of X is 


Please choose your answer from the right side options

1.6 

0.24 

0.84 

0.75

152017318

If \(A=\begin{bmatrix} 1 &0 &1 \\ 0& 2& 0\\ 1&-1 &4 \end{bmatrix},A=B+C,B=B^{T}\,\) and \(C=-C^{T}\,\) ,  then \(C= \)


Please choose your answer from the right side options

\(\begin{bmatrix} 0 &0.5 & 0\\ -0.5 &0 &0 \\ 0 & 0 & 0 \end{bmatrix} \)

\(\begin{bmatrix} 0 &0.5 & 0\\ 0 &0 &0.5 \\ 0 & -0.5 & 0 \end{bmatrix} \)

\(\begin{bmatrix} 0 &-0.5 & 0.5\\ 0.5 &0 &0 \\ -0.5 & 0 & 0 \end{bmatrix} \)

\(\begin{bmatrix} 0 &0.5 & 0\\ -0.5 &0 &0.5 \\ 0 & -0.5 & 0 \end{bmatrix}\)

152017319

If \(\vec{a} \) is a unit vector, then \(\left |\vec{a}\times \hat{i} \right |^2+\left |\vec{a}\times \hat{j} \right |^2+\left |\vec{a}\times \hat{k} \right |^2=\) 


Please choose your answer from the right side options

1

0

152018311

 Let \(f:\mathbb{R}\rightarrow \mathbb{R},\,g:\mathbb{R}\rightarrow \mathbb{R}\) be differentiable function such that \((fog)(x)=x\)  . If \(f(x)=2x+cosx+sin^2x\) , then the value of \( \sum_{n=1}^{99}g(1+(2n-1)\pi )\) is


Please choose your answer from the right side options

\(1250\pi\)

\((99)^2\frac{\pi}{2}\)

\((99)^2{\pi}\)

\(2500\pi\)

152018312

If \(f:[1,\infty )\rightarrow [1,\infty ]\) is defined by \(f(x)=\frac{1+\sqrt{1+4log_2\,x}}{1}\) then \( \,\,f^{-1}(3)=\)


Please choose your answer from the right side options

\(0\)

\(1\)

\(64\)

\(\frac{1+\sqrt{5}}{2}\)

152018313

If \(α\) and \(β\) are the greatest divisors of \(n(n^2-1)\) and \(2n(n^2+2)\) respectively for all \(n\,\epsilon\, N\) then \(αβ=\)


Please choose your answer from the right side options

\(18\)

\(36 \)

\(27 \)

\(9\)

152018314

Let \(A=\begin{bmatrix} \frac{1}{6} & \frac{-1}{3} &\frac{-1}{6} \\ \frac{-1}{3}& \frac{2}{3} &\frac{1}{3} \\ \frac{-1}6{} & \frac{1}{3} & \frac{1}{6} \end{bmatrix}\) . If \(A^{2016l}+A^{2017m}+A^{2018n}=\frac{l}{\alpha }A \,\, \) for every \(l,m,n\,\epsilon\, N\) , then the value of \(α\) is


Please choose your answer from the right side options

\(\frac{1}{6}\)

\(\frac{1}{3}\)

\(\frac{1}{2}\)

\(\frac{2}{3}\)

152018315

Let \(l,m,n\,\epsilon\, \mathbb{R}\) and \(A=\begin{bmatrix} 1 & r& r^2 &1 \\ r& r^2 &1 & m\\ r^2& 1 &r & n \end{bmatrix}\) . Then the set of all real values of r for which the rank of A is 3, is


Please choose your answer from the right side options

\((0,\infty)\)

\(R\)

\(R-\left \{ 1 \right \}\)

\(R-\left \{ 0 \right \}\)

152018316

The following system of equations

 \(x+y+z=9 \)

\(2x+5y+7z=52\)

\(x+7y+11z=77\)

has


Please choose your answer from the right side options

no solution 

exactly 2 solutions 

only one solution 

infinitely many solutions

152018317

\(Z\) is a complex number such that \(\left | Z \right |\leq 2\) and \( \,\,-\frac{\pi }{3}\leq \,amp\,\,Z\leq \frac{\pi }{3}\) .The area of the region formed by locus of \(Z\) is


Please choose your answer from the right side options

\(\frac{2\pi}{3}\)

\(\frac{\pi}{3}\)

\(\frac{4\pi}{3}\)

\(\frac{8\pi}{3}\)

152018318

The points on the argand plane is given by \(Z_1=-3+5i,\,Z_2=-1+6i,\,Z_3=-2+8i,\,Z_4=-4+7i\) form a


Please choose your answer from the right side options

parallelogram 

rectangle 

rhombus 

square

152018319

When \(n=8,\,(\sqrt{3}+i)^n+(\sqrt{3}-i)^n=\)


Please choose your answer from the right side options

\(−256 \)

\(−128 \)

\(256i \)

\(128i\)

152019311

The domain of the function \(\Large f(x)= sin^{-1} [log_4 \ (\frac{x}{4})] + \sqrt{17x - x^2-16}\) is


Please choose your answer from the right side options

[−1, 1]

[1, 4]

[0, 16]

[1, 16]

152019312

If \(\large f:[1,\infty) \longrightarrow [0,\infty)\) is given by \(\Large f(x) = x-\frac{1}{x}\)then \(\large f^{-1}(x)=\)


Please choose your answer from the right side options

\(\large x + \sqrt{x^2 + 4}\)

\(\Large \frac{x}{x^2 -1}\)

\(\Large \frac{1}{2}[x + \sqrt{x^2 + 4}]\)

\(\Large \frac{1}{2}[x - \sqrt{x^2 + 4}]\)

152019313

If greatest divisor of \(\large 30.5^{2n} + 4.2^{3n}\) is p, \(\large ∀ ∈ N\) and \(\large 2^{2n+1} - 6n -2\) is q, \(\large ∀ ∈ N\), then p + q =


Please choose your answer from the right side options

26

52

104

13

152019314

If \(\large A=\begin{bmatrix} 0 & 5 \\ 0 & 0 \\ \end{bmatrix}\) and \(\large f(x) = x+x^2 +...+ \ x^{2018}\), the f(A) + I =


Please choose your answer from the right side options

\(\large \begin{bmatrix} 0 & 0 \\ 0 & 0 \\ \end{bmatrix}\) 

\(\large \begin{bmatrix} 1 & 5 \\ 0 & 0 \\ \end{bmatrix}\) 

\(\large \begin{bmatrix} 0 & 5 \\ 1 & 1 \\ \end{bmatrix}\) 

\(\large \begin{bmatrix} 1 & 5 \\ 0 & 1 \\ \end{bmatrix}\) 

152019315

If a, b, c are real numbers such that a−b= 1, b-c = 3 then the number of matrices of the form\(\large A= \begin{bmatrix} 1 & 1 & 1\\ a & b &c \\ a^2 & b^2 & c^2\\ \end{bmatrix}\) such that |A| = -12 is


Please choose your answer from the right side options

1

2

3

Infinitely

152019316

For what values of ‘a’ the system of equations x + y + z = 1, 2x + 3y + 2z =2, ax + ay + 2az = 4 will have a unique solution? 


Please choose your answer from the right side options

For a = 0 only.

For \(\large a \ \epsilon R - \{0\}\)

For all \(\large a \ \epsilon \ \mathbb{Q}\)

For all \(\large a \notin \mathbb{N}\)

152019317

If \(\large z_n =(1+i\sqrt 2)^{n}, n \ \epsilon \ \mathbb{Z}\), then \(\Large \frac{1}{9} Re(z_4 \bar z_5) = \)


Please choose your answer from the right side options

81

27

9

3

152019318

If z = x + iy, then the centre of circle \(\Large |\frac{z- 3}{z-2i} | = 2\) is


Please choose your answer from the right side options

\(\Large (-1, -\frac{8}{3})\)

\(\Large (1, \frac{8}{3})\)

\(\Large (-1, \frac{8}{3})\)

\(\Large (1, -\frac{8}{3})\)

152019319

If z is a complex number such that \(\large |z + 4| \geq 3,\) then the smallest value of \(\large |z+3|\) is


Please choose your answer from the right side options

3

1

2

0

152020311

Match the items of List-I with those of the items of List-II

List-I List-II
A) Range of \(Sec^{-1}[1+cos^2x],[.] \) denote greatest integer function I) odd function
B) Domain of f(x), where \(f\left ( x+{\Large\frac{1}{x}} \right ) =x^2+{\Large\frac{1}{x^2}} \) II)\(\left\{ 0,\Large\frac{1}{2}\right\}\)
C) \(f(x+y)=f(x)+f(y) :f(1)=5\) III)\(\left\{ Sec^{-1}5,Sec^{-1}4\right\} \)
D)\(Sin^{-1}x-Cos^{-1}x+Sin^{-1}(1-x)=0\Rightarrow x\,\epsilon\) IV) \(\mathbb{R}-(-2,2)\)
  V) \(\left\{ {Sec^{-1}1 ,Sec^{-1}2}\right\}\)

 


Please choose your answer from the right side options

A B C D

V IV I II

A B C D

III IV II I

A B C D

V II III IV

A B C D

III II I IV

152020312

The domain of the function \(f(x)=Sec^{-1}(3x-4)+Tanh^{-1}\left ( {\Large\frac{x+3}{5}} \right ) \) is


Please choose your answer from the right side options

\((-8,1)\cup \left ( {\Large\frac{5}{3}},2 \right ) \)

\(\left (1, {\Large\frac{5}{3}} \right ) \)

\([-8,1] \cup \left [ {\Large\frac{5}{3}},2 \right ]\)

\((-8,1]\cup \left [ {\Large\frac{5}{3}},2 \right )\)

152020313

Let \(m = (9n^2 + 54n + 80) ( 9n^2 + 45n + 54) ( 9n^2 + 36n + 35)\). The greatest positive integer which divides m, for all positive integers n , is 


Please choose your answer from the right side options

720 

724 

696 

842

152020314

If A is a 3×3 matrix and the matrix obtained by replacing the elements of A with their corresponding cofactors is \(\begin{bmatrix} 1& -2 & 1\\ 4& -5& -2 \\ -2& 4&1 \\ \end{bmatrix}\) , then a possible value of the determinant of A  is


Please choose your answer from the right side options

1

152020315

If \(A=\begin{bmatrix} 3 & -3 & 4 \\ 2 & -3 & 4 \\ 0 & -1 & 1 \\ \end{bmatrix} \) then \(\left ( A^2 \right )^{-1} \) =


Please choose your answer from the right side options

\(A^2\) 

2A 

\(A^3\)

\( A=\begin{bmatrix} 1 & 2 & 2 \\ 2 & 1 & -2 \\ -2 & 2 & -1 \\ \end{bmatrix}\)

152020316

The equations x + y + z = 3, x + 2y + 2z = 6 and x + ay +3z = b have 


Please choose your answer from the right side options

No solution when a ≠ 3 , b is any value 

Infinite number of solutions when b ≠ 9 

Unique solution when a ≠ 3, b is any value 

Unique solution when a = 3 and b ≠ 9

152020317

Let \(Z\,\epsilon\,\mathbb{C}\) and \(i=\sqrt{-1} \), if \(a,b,c\,\epsilon (0,1) \) be such that \(a^2+b^2+c^2=1 \) and \(b+ic=(1+a)z\)  , then  \(\Large\frac{1+iz}{1-iz}\)=


Please choose your answer from the right side options

\(\Large\frac{a+ib}{1+c} \)

\(\Large\frac{a-ib}{1+c} \)

\(\Large\frac{a-ib}{1-c} \)

\(\Large\frac{a+ib}{1-c}\)

152020318

If \(A=\begin{Bmatrix} z=x+iy/\,real\,\,part\,\,of\,\,{\Large\frac{\bar{z}-1}{z-i}}=2\end{Bmatrix}\), then the locus of the point \(P( x, y)\) in the cartesian plane is


Please choose your answer from the right side options

a pair of lines passing through (-1, -1) 

a circle of radius \(\Large\frac{1}{\sqrt2}\) and the centre \(\left (\Large \frac{-1}{2},\frac{3}{2} \right )\)

a pair of lines passing through (-1, -2) 

a circle of radius \(\Large\frac{1}{2}\)

152020319

If \(ω\) is a complex cube root of unity ,then \(\left ( 1-\omega +\omega^2 \right )^6+\left ( 1-\omega^2+\omega \right )^6\) =


Please choose your answer from the right side options

64 

128

162019311

Matrix \(A_r= \begin{bmatrix} r & r-1\\ r-1 & r \end{bmatrix} ; r=1,2,3,.....\) If  \( \sum_{r=1}^{100}\begin{vmatrix} A_r \end{vmatrix}=(\sqrt{10})^k\) , then \(K=..........;(\left | A_r \right |=det(A_r))\)


Please choose your answer from the right side options

\(2\)

\(6\)

\(4\)

\(8\)

162019312

\(\frac{d}{dx}\left ( 3cos\left ( \frac{\pi }{6}+x^\circ \right ) -4cos^3\left ( \frac{\pi}{6} +x^\circ\right )\right )=................\)


Please choose your answer from the right side options

\(cos(3x^\circ)\)

\(\frac{\pi}{60}sin(3x^\circ)\)

\(\frac{\pi}{60}cos(3x^\circ)\)

\(-\frac{\pi}{60}sin(3x^\circ)\)

162019313

If  \(f(x)=1+x+x^2+.......x^{1000}\)   , then \( f'(–1) = …….\)


Please choose your answer from the right side options

\( –50 \)

\(–500 \)

\( –100 \)

\(500500 \)

162019314

Applying mean value theorem on \(f(x)=logx;\,x\,\varepsilon\, [1,e]\) the value of c = ………


Please choose your answer from the right side options

\(log(e-1)\)

\(e-1\)

\(1-e\)

\(2\)

162019315

If  \(\int sin^{13}\,xcos^3\,xdx=Asin^{14}x+Bsin^{16}x+C\) , then A + B = ……..


Please choose your answer from the right side options

\(\frac{1}{110}\)

\(\frac{17}{112}\)

\(\frac{15}{112}\)

\(\frac{1}{112}\)

162019316

If \(\int \frac{1+cosx}{cosx-cos^2x}dx=log\left | secx+tanx \right |-2f'(x)+C\) ,  then f(x) = ……..


Please choose your answer from the right side options

\(2\,cot\left ( \frac{x}{2} \right )\)

\(2\,log\left | sin\frac{x}{2} \right |\)

\(-2\,cot\left ( \frac{x}{2}\right )\)

\(-2\,log\left | sin\frac{x}{2} \right |\)

162019317

The probability that an event A occurs in a single trial of an experiment is \(0.6\) . In the first three independent trials of the experiment, the probability that A occurs atleast once is ……….


Please choose your answer from the right side options

\( 0.930 \)

\( 0.936\)

\( 0.925 \)

\( 0.927\)

162019318

If \(6P(A)=8P(B)=14P(A\cap B)=1\), then the \(P\left ( \frac{A'}{B} \right )=.........\)


Please choose your answer from the right side options

\(\frac{3}{7}\)

\(\frac{4}{7}\)

\(\frac{3}{5}\)

\(\frac{2}{5}\)

162019319

The mean and variance of a random variable X having a binomial distribution are 6 and 3 respectively. The probability of variable X less than 2 is


Please choose your answer from the right side options

\(\frac{13}{2048}\)

\(\frac{13}{4096}\)

\(\frac{15}{4096}\)

\(\frac{25}{2048}\)

172020311

\(S_n\) denotes the sum of n terms of an AP, whose first term is a. If the common difference \(d = S_n– k \,\,S_{n-1} + S_{n–2}\) , then k is equal to


Please choose your answer from the right side options

7

172020312

If \(Z_1 \) and \(Z_2 \) are two complex numbers such that \(\left|Z_1 \right| =\left|Z_2 \right| \) and \(arg(Z_1) +arg(Z_2) =\pi \), then \(Z_1 \) is equal to


Please choose your answer from the right side options

\(2\bar{Z_2} \)

\(\bar{Z_2} \)

\(-\bar{Z_2}\) 

None of these

172020313

If \(Z_1, Z_2\) and \(Z_3\) represent the vertices of an equilateral triangle such that \(|Z_1| = |Z_2| = |Z_3|\), then 


Please choose your answer from the right side options

\(Z_1 + Z_2 = Z_3\)

\(Z_1 + Z_2 + Z_3 = 0\) 

\(Z_1 Z_2 = Z_3\) 

\(Z_1 – Z_2 = Z_3 – Z_2\)

172020314

If the equation \(x^2+2x+3=0\) and \(ax^2+bx+c=0\), a, b, c ∈ R, have a common root, then a:b:c is


Please choose your answer from the right side options

3 : 2 : 1 

1:3:2 

3:1:2 

1:2:3

172020315

If a, b, c are in GP and \(a^{\Large\frac{1}{x}}=b^{\Large\frac{1}{y}}=c^{\Large\frac{1}{z}}\) , then x, y, z are in 


Please choose your answer from the right side options

AP 

GP 

HP 

None of these

172020316

If p, q, r and s are positive real numbers such that p + q + r + s = 2, then M = (p+q) (r+s) satisfies the relation, when 


Please choose your answer from the right side options

0 < M \(\leq\)

1 \(\leq\) M \(\leq\) 2 

2 \(\leq\) M \(\leq\)

3 \(\leq\) M \(\leq\) 4

172020317

The sum of the infinite series \({\Large\frac{2^2}{2!}}+{\Large\frac{2^4}{4!}}+{\Large\frac{2^6}{6!}}+...... \) is


Please choose your answer from the right side options

\({\Large\frac{e^2+1}{2}} \)

\({\Large\frac{e^4+1}{2e^2}} \)

\({\Large\frac{(e^2-1)^2}{2e^2}} \)

\({\Large\frac{(e^2+1)^2}{2e^2}} \)

172020318

If n is a positive integer, then \(n^3 + 2n\) is divisible by 


Please choose your answer from the right side options

15 

3

172020319

If a and b are the coefficients of \(x^r\) and \(x^{n–r}\) respectively in the expansion of \((1+x)^n\), then 


Please choose your answer from the right side options

a = b 

\(a + b = n^2\) 

a = nb 

a – b = n

1020183110

If \(f\) is differentiable at \(x=1\) , then \(\lim_{x\rightarrow 1}\frac{x^2f(1)-f(x)}{x-1}\) is


Please choose your answer from the right side options

'>

\(-f'(1)\)

'>

\(f(1)-f'(1)\)

'>

\(2f(1)-f'(1)\)

'>

\(2f(1)+f'(1)\)

'>

\(f(1)+f'(1)\)

1020183111

Eccentricity of the ellipse \(4x^2+y^2-8x+4y-8=0\) is


Please choose your answer from the right side options

\(\frac{\sqrt3}{2}\)

\(\frac{\sqrt3}{4}\)

\(\frac{\sqrt3}{\sqrt2}\)

\(\frac{\sqrt3}{8}\)

\(\frac{\sqrt3}{16}\)

1020183112

The focus of the parabola \((y+1)^2=-8(x+2)\) is


Please choose your answer from the right side options

\((-4,-1)\)

\((-1,-4)\)

\((1,4)\)

\((4,1)\)

\((-1,4)\)

1020183113

Which of the following is the equation of a hyperbola?


Please choose your answer from the right side options

\(x^2-4x+16y+17=0\)

\(4x^2+4y^2-16x+4y-60=0\)

\(x^2+2y^2+4x+2y-27=0\)

\(x^2-y^2+3x-2y-43=0\)

\(x^2+4x+6y-2=0\)

1020183114

Let \(f(x)=px^2+qx+r\) , where \(p,q,r\) are constants and \(p\neq0\) If  \(f(5)=-3f(2)\) and \(f(-4)=0\) , then the other root of \(f\) is


Please choose your answer from the right side options

\(3\)

\(-7\)

\(-2\)

\(2\)

\(6\)

1020183115

Let \(f:R\rightarrow R\) satisfy \(f(x)f(y)=f(xy)\) for all real numbers \(x\) and \(y\) . If \(f(2)=4\) , then \(f(\frac{1}{2})=\)


Please choose your answer from the right side options

\(0\)

\(\frac{1}{4}\)

\(\frac{1}{2}\)

\(1\)

\(2\)

1020183116

Sum of last \(30\) coefficents in the binomial expansion of \((1+x)^{59}\) is


Please choose your answer from the right side options

\(2^{29}\)

\(2^{59}\)

\(2^{58}\)

\(2^{59}-2^{29}\)

\(2^{60}\)

1020183117

\((\sqrt{3}+\sqrt{2})^4-(\sqrt{3}-\sqrt{2})^4=\)


Please choose your answer from the right side options

\(20\sqrt6\)

\(30\sqrt6\)

\(5\sqrt{10}\)

\(40\sqrt6\)

\(10\sqrt6\)

1020183118

Three players \(A,B\) and \(C\) play a game. The probability that \(A,B\) and \(C\) will finish the game are respectively \(\frac{1}{2},\frac{1}{3}\) and \(\frac{1}{4}\) . The probability that the game is finished is.


Please choose your answer from the right side options

\(\frac{1}{8}\)

\(1\)

\(\frac{1}{4}\)

\(\frac{3}{4}\)

\(\frac{1}{2}\)

1020183119

If \(z_1=2-i\) and \(z_2=1+i\) , then \(\large \left | \frac{z_1+z_2+1}{z_1-z_2+i} \right |\) is


Please choose your answer from the right side options

\(2\)

\(2\sqrt2\)

\(3\)

\(\sqrt3\)

None of these

1020183120

If \(f(x)=\large \sqrt{\frac{x-sinx}{x+cos^2x}}\), then \(\lim_{x\rightarrow \infty }f(x)\) is equal to


Please choose your answer from the right side options

\(1\)

\(2\)

\(\frac{1}{2}\)

\(0\)

\(\infty\)

1020183121

The value of \(sin\frac{31}{3}\pi\) is


Please choose your answer from the right side options

\(\frac{\sqrt3}{2}\)

\(\frac{1}{\sqrt2}\)

\(\frac{-\sqrt3}{2}\)

\(\frac{-1}{\sqrt2}\)

\(\frac{1}{2}\)

1020183122

The sum of odd integers from \(1\) to \(2001\) is


Please choose your answer from the right side options

\((1121)^2\)

\((1101)^2\)

\((1001)^2\)

\((1021)^2\)

\((1011)^2\)

1020183123

If  \(y=\frac{sin^2x}{1+cotx}+\frac{cos^2x}{1+tanx}\) , then \(y'(x)\) is equal to


Please choose your answer from the right side options

\(2cos^2x\)

\(2cos^3x\)

\(-cos2x\)

\(cos2x\)

\(3cosx\)

1020183124

The foci of the hyperbola \(16x^2-9y^2-64x+18y-90=0\) are


Please choose your answer from the right side options

\(\left ( \frac{24\pm 5\sqrt{145}}{12} ,1\right )\)

\(\left ( \frac{21\pm 5\sqrt{145}}{12} ,1\right )\)

\(\left ( 1,\frac{24\pm 5\sqrt{145}}{2} \right )\)

\(\left ( 1,\frac{21\pm 5\sqrt{145}}{2} \right )\)

\(\left ( \frac{21\pm 5\sqrt{145}}{2} ,-1\right ) \)

1020183125

If the sum of the coefficients in the expansions of \((a^2x^2-2ax+1)^{51}\) is zero, then \(a\) is equal to


Please choose your answer from the right side options

\(0\)

\(1\)

\(-1\)

\(-2\)

\(2\)

1020183126

The mean deviation of the data \(2,9,9,3,6,9,4\) from the mean is


Please choose your answer from the right side options

\(2.23\)

\(3.23\)

\(2.57\)

\(3.57\)

\(1.03\)

1020183127

The mean and variance of a binomial distribution are \(8\) and \(4\) respectively. What is \((X=1)\) ?


Please choose your answer from the right side options

\(\frac{1}{2^8}\)

\(\frac{1}{2^{12}}\)

\(\frac{1}{2^6}\)

\(\frac{1}{2^4}\)

\(\frac{1}{2^5}\)

1020183128

The number of diagonals of a polygon with \(15\) sides is


Please choose your answer from the right side options

\(90\)

\(45\)

\(60\)

\(70\)

\(10\)

1020183129

In a class, \(40\%\) of students study Maths and Science and \(60\%\) of students study Maths. What is the probability of a students studying Science given the student is already studying Maths?


Please choose your answer from the right side options

\(\frac{1}{3}\)

\(\frac{1}{6}\)

\(\frac{2}{3}\)

\(\frac{1}{5}\)

\(\frac{1}{4}\)

1020183130

The eccentricity of the conic \(x^2+2y^2-2x+3y+2=0\) is


Please choose your answer from the right side options

\(0\)

\(\frac{1}{\sqrt2}\)

\(\frac{1}{2}\)

\(\sqrt2\)

\(1\)

1020183131

If the mean of a set of observations \(x_1,x_2,...x_{10}\) is \(20\) , then the mean of \(x_1+4,x_2+8,x_3+12,...x_{10}+40\) is


Please choose your answer from the right side options

\(34\)

\(32\)

\(42\)

\(38\)

\(40\)

1020183132

A letter is taken at random from the word “STATISTICS” and another letter is taken at random from the word “ASSISTANT”. The probability that they are same letters is


Please choose your answer from the right side options

\(\frac{1}{45}\)

\(\frac{13}{90}\)

\(\frac{19}{90}\)

\(\frac{5}{18}\)

\(\frac{9}{10}\)

1020183133

If \(sin\,\alpha\) and \(cos\,\alpha\) are the roots of the equation \(ax^2+bx+c=0\) , then


Please choose your answer from the right side options

\(a^2-b^2+2ac=0\)

\((a-c)^2=b^2+c^2\)

\(a^2+b^2-2ac=0\)

\(a^2+b^2+2ac=0\)

\(a+b+c=0\)

1020183134

If the sides of triangle are \(4,5\) and \(6\) cm. Then the area (in sq cm) of triangle is


Please choose your answer from the right side options

\(\frac{\pi}{4}\)

\(\frac{\pi}{4} \sqrt7\)

\(\frac{4}{15}\)

\(\frac{4}{15}\sqrt7\)

\(\frac{15}{4}\sqrt7\)

1020183135

In a group of \(6\) boys and \(4\) girls, a team consisting of four children is formed such that the team has atleast one boy. The number of ways of forming a team like this is


Please choose your answer from the right side options

\(159\)

\(209\)

\(200\)

\(240\)

\(212\)

1020183136

A password is set with \(3\) distinct letters from the word LOGARITHMS. How many such passwords can be formed?


Please choose your answer from the right side options

\(90\)

\(720\)

\(80\)

\(72\)

\(120\)

1020183137

If \(5^{97}\) is divided by \(52\) , the remainder obtained is


Please choose your answer from the right side options

\(3\)

\(5\)

\(4\)

\(0\)

\(1\)

1020183138

A quadratic equation \(ax^2+bx+c=0\) , with distinct coefficients is formed. It \(a,b,c\) are chosen from the numbers \(2,3,5\) then the probability that the equation has real roots is 


Please choose your answer from the right side options

\(\frac{1}{3}\)

\(\frac{2}{5}\)

\(\frac{1}{4}\)

\(\frac{1}{5}\)

\(\frac{2}{3}\)

1020183139

 \(\lim_{x\rightarrow \infty }\frac{3x^3+2x^2-7x+9}{4x^3+9x-2}\) is equal to


Please choose your answer from the right side options

\(\frac{2}{9}\)

\(\frac{1}{2}\)

\(\frac{-9}{2}\)

\(\frac{3}{4}\)

\(\frac{9}{2}\)

1020183140

The minimum value of  \(f(x)=max\left \{ x,1+x,2-x \right \}\) is


Please choose your answer from the right side options

\(\frac{1}{2}\)

\(\frac{3}{2}\)

\(1\)

\(0\)

\(2\)

1020183141

The equations of the asymptotes of the hyperbola \(xy+3y-2y-10=0\) are


Please choose your answer from the right side options

\(x=-2,y=-3\)

\(x=2,y=-3\)

\(x=2,y=3\)

\(x=4,y=3\)

\(x=3,y=4\)

1020183142

If \(f(x)=x^6+6^x\), then \(f'(x)\) is equal to


Please choose your answer from the right side options

\(12x\)

\(x+4\)

\(6x^5+6^xlog(6)\)

\(6x^5+x6^{x-1}\)

\(x^6\)

1020183143

The standard deviation of the data \(6,5,9,13,12,8,10\) is


Please choose your answer from the right side options

\(\frac{\sqrt{52}}{7}\)

\(\frac{52}{7}\)

\(\frac{\sqrt{53}}{7}\)

\(\frac{53}{7}\)

\(6\)

1020183144

\(\lim_{x\rightarrow 0}\frac{1-cos\,mx}{1-cos\,nx}=\)


Please choose your answer from the right side options

\(\frac{m^2}{n^2}\)

\(\frac{n^2}{m^2}\)

\(\infty\)

\(-\infty\)

\(0\)

1020183145

\(\lim_{x\rightarrow 0}\frac{(\sqrt{1+2x})-1}{x}=\)


Please choose your answer from the right side options

\(0\)

\(-1\)

\(\frac{1}{2}\)

\(1\)

\(\frac{-1}{2}\)

1020183146

Let \(f\) and \(g\) be differentiable functions such that  \(f(3)=5,g(3)=7,f'(3)=13,g'(3)=6,f'(7)=2\,\,and\,\,g'(7)=0\) . If  \(h(x)=(fog)(x)\) , then \(h'(3)=\)


Please choose your answer from the right side options

\(14\)

\(12\)

\(16\)

\(0\)

\(10\)

1020183147

\(\frac{\sqrt{3}}{sin(20^\circ)}-\frac{1}{cos(20^\circ)}=\)


Please choose your answer from the right side options

\(1\)

\(\frac{1}{\sqrt2}\)

\(2\)

\(4\)

\(0\)

1020183148

A poison variate \(X\) satisfies \(P(X=1)=P(X=2).P(X=6)\) is equal to


Please choose your answer from the right side options

\(\frac{4}{45}\,e^{-2}\)

\(\frac{1}{45}\,e^{-1}\)

\(\frac{1}{9}\,e^{-2}\)

\(\frac{1}{4}\,e^{-2}\)

\(\frac{1}{45}\,e^{-2}\)

1020183149

Let \(a\) and \(b\) be \(2\) consecutive integers selected from the first \(20\) natural numbers. The probability that \(\sqrt{a^2+b^2+a^2b^2}\) is an odd positive integer is


Please choose your answer from the right side options

\(\frac{9}{19}\)

\(\frac{10}{19}\)

\(\frac{13}{19}\)

\(1\)

\(0\)

1020183150

An ellipse of eccentricity \(\frac{2\sqrt2}{3}\) is inscribed in a circle. A point is chosen inside the circle at random. The probability that the point lies outside the ellipse is


Please choose your answer from the right side options

\(\frac{1}{3}\)

\(\frac{2}{3}\)

\(\frac{1}{9}\)

\(\frac{2}{9}\)

\(\frac{1}{27}\)

1020183151

If the vectors \(4\hat{i}+11\hat{j}+m\hat{k},7\hat{i}+2\hat{j}+6\hat{k}\,\,and\,\,\hat{i}+5\hat{j}+4\hat{k}\) are coplanar, then \(m\) is equal to


Please choose your answer from the right side options

\(38\)

\(0\)

\(10\)

\(-10\)

\(25\)

1020183152

Let  \(\vec{a}=\hat{i}+\hat{j}+\hat{k},\vec{b}=\hat{i}+3\hat{j}+5\hat{k}\,\,and\,\,\vec{c}=7\hat{i}+9\hat{j}+11\hat{k}\) .  Then, the area of the parallelogram with diagonals \(\vec{a}+\vec{b}\) and  \(\vec{b}+\vec{c}\) is


Please choose your answer from the right side options

\(4\sqrt6\)

\(\frac{1}{2}\sqrt{21}\)

\(\frac{\sqrt6}{2}\)

\(\sqrt6\)

\(\frac{1}{\sqrt6}\)

1020183153

If \(\left | \vec{a} \right |=3,\left | \vec{b} \right |=1,\left | \vec{c} \right |=4\) and  \(\vec{a}+\vec{b}+\vec{c}=0\) , then the value of  \(\vec{a}\cdot \vec{b}+\vec{b}\cdot \vec{c}+\vec{c}\cdot \vec{a}\)  is equal to


Please choose your answer from the right side options

\(13\)

\(26\)

\(-29\)

\(-13\)

\(-26\)

1020183154

If \(\left | \vec{a}-\vec{b} \right |=\left | \vec{a} \right |=\left | \vec{b} \right |=1\) , then the angle between \(\vec{a}\) and  \(\vec{b}\) is equal to


Please choose your answer from the right side options

\(\frac{\pi}{3}\)

\(3\frac{\pi}{4}\)

\(\frac{\pi}{2}\)

\(0\)

\(\pi\)

1020183155

If the vectors \(\vec{a}=\hat{i}-\hat{j}+2\hat{k},\vec{b}=2\hat{i}+4\hat{j}+\hat{k}\,\,and\,\,\vec{c}=\lambda \hat{i}+9\hat{j}+\mu \hat{k}\) are mutually orthogonal, then \(\lambda+\mu\) is  equal to


Please choose your answer from the right side options

\(5\)

\(-9\)

\(-1\)

\(0\)

\(-5\)

1020183156

The solution of  \(x^{\frac{2}{5}}+3x^{\frac{1}{5}}-4=0\)  are 


Please choose your answer from the right side options

\(1,1024\)

\(-1,1024\)

\(1,1031\)

\(-1024,1\)

\(-1,1031\)

1020183157

If the equations \(x^2+ax+1=0\) and \(x^2-x-a=0\) have a real common root \(b\) , then the value of \(b\) is equal to


Please choose your answer from the right side options

\(0\)

\(1\)

\(-1\)

\(2\)

\(3\)

1020183158

If \(sin\,\theta-cos\,\theta=1\) , then the value of \(sin^3\,\theta-cos^3\,\theta\) is equal to


Please choose your answer from the right side options

\(1\)

\(-1\)

\(0\)

\(2\)

\(-2\)

1020183159

Two dice of different colours are thrown at a time. The probability that the sum is either \(7\) or \(11\) is


Please choose your answer from the right side options

\(\frac{7}{36}\)

\(\frac{2}{9}\)

\(\frac{2}{3}\)

\(\frac{5}{9}\)

\(\frac{6}{7}\)

1020183160

\(\frac{1}{9!}+\frac{1}{3!7!}+\frac{1}{5!5!}+\frac{1}{7!3!}+\frac{1}{9!}\) Is equal to


Please choose your answer from the right side options

\(\frac{2^9}{10!}\)

\(\frac{2^{10}}{8!}\)

\(\frac{2^{11}}{9!}\)

\(\frac{2^{10}}{7!}\)

\(\frac{2^8}{9!}\)

1020183161

The order and degree of the differential equation \((y''')^2+(y'')^3-(y')^4+y^5=0\) is


Please choose your answer from the right side options

\(3\,and\,2\)

\(1\,and\,2\)

\(2\,and\,3\)

\(1\,and\,4\)

\(3\,and\,5\)

1020183162

\(\int_{-2}^{2}\left | x \right |dx\) is equal to


Please choose your answer from the right side options

\(0\)

\(1\)

\(2\)

\(4\)

\(\frac{1}{2}\)

1020183163

\(\int_{-1}^{0}\frac{dx}{x^2+2x+2}\) is equal to


Please choose your answer from the right side options

\(0\)

\(\frac{\pi}{4}\)

\(\frac{-\pi}{4}\)

\(\frac{\pi}{2}\)

\(\frac{-\pi}{2}\)

1020183164

If \(\int_{-1}^{4}f(x)dx=4\) and \(\int_{2}^{4}(3-f(x))dx=7\) , then \(\int_{-1}^{2}f(x)dx\) is


Please choose your answer from the right side options

\(1\)

\(2\)

\(3\)

\(4\)

\(5\)

1020183165

\(\int \frac{xe^x}{(1+x)^2}dx=\)


Please choose your answer from the right side options

\( \frac{e^x}{1+x}+C\)

\( \frac{e^x}{1+e^x}+C\)

\( \frac{e^{2x}}{1+e^x}+C\)

\( \frac{e^{-x}}{1+x}+C\)

\( \frac{e^{-2x}}{1+x}+C\)

1020183166

The remainder when \(2^{2000}\) is divided by \(7\) is


Please choose your answer from the right side options

\(1\)

\(2\)

\(8\)

\(12\)

\(4\)

1020183167

The coefficient of  \(x^5\) in the expansion of \((x+3)^8\) is


Please choose your answer from the right side options

\(1542\)

\(1512\)

\(2512\)

\(12\)

\(4\)

1020183168

The maximum value of \(5\,cos\,\theta+3\,cos\left ( \theta+\frac{\pi}{3} \right )+3\)  is


Please choose your answer from the right side options

\(5\)

\(11\)

\(10\)

\(-1\)

\(2\)

1020183169

The area of the triangle in the complex plane formed by \(z,iz\) and \(z+iz\) is


Please choose your answer from the right side options

\(\left | z \right |\)

\(\left | \bar{z} \right |^2\)

\(\frac{1}{2}\left | {z} \right |^2\)

\(\frac{1}{2}\left | {z+iz} \right |^2\)

\(\left | {z+iz} \right |\)

1020183170

Let \(f:f(-x)\rightarrow f(x)\) be a differentiable function. If \(f\) is even, then \(f'(0)\) is equal to


Please choose your answer from the right side options

\(1\)

\(2\)

\(0\)

\(-1\)

\(\frac{1}{2}\)

1020183171

The coordinate of the point dividing internally the line joining the points \((4,-2)\) and \((8,6)\) in the ratio \(7:5\) is


Please choose your answer from the right side options

\((16,18)\)

\((18,16)\)

\((\frac{19}{3},\frac{8}{3})\)

\((\frac{8}{3},\frac{19}{3})\)

\((7,3)\)

1020183172

The area of the triangle formed by the points \((a,b+c),(b,c+a),(c,a+b)\) is


Please choose your answer from the right side options

\(abc\)

\(a^2+b^2+c^2\)

\(ab+bc+ca\)

\(0\)

\(a(ab+bc+ca)\)

1020183173

If \((x,y)\) is equidistant from \((a+b,b-a)\) and \((a-b,b+a)\), then


Please choose your answer from the right side options

\(ax+by=0\)

\(ax-by=0\)

\(bx+ay=0\)

\(bx-ay=0\)

\(x=y\)

1020183174

The equation of the line passing through \((a,b)\) and parallel to the line \(\frac{x}{a}+\frac{y}{b}=1\) is


Please choose your answer from the right side options

\(\frac{x}{a}+\frac{y}{b}=3\)

\(\frac{x}{a}+\frac{y}{b}=2\)

\(\frac{x}{a}+\frac{y}{b}=0\)

\(\frac{x}{a}+\frac{y}{b}+2=0\)

\(\frac{x}{a}+\frac{y}{b}=4\)

1020183175

If the points \((2a,a),(a,2a)\) and \((a,a)\) enclose a triangle of area \(18\,sq\) units, then the centroid of the triangle is equal to


Please choose your answer from the right side options

\((4,4)\)

\((8,8)\)

\((-4,-4)\)

\((4\sqrt{2},4\sqrt{2})\)

\((6,6)\)

1020183176

The area of a triangle is \(5\,sq\) units. Two of its vertices are \((2,1)\) and \((3,-2)\) . The third vertex lies on \(y=x+3\) . The coordinates of the third vertex can be


Please choose your answer from the right side options

\(\left (\frac{-3}{2}, \frac{-3}{2} \right )\)

\(\left (\frac{3}{4}, \frac{-3}{2} \right )\)

\(\left (\frac{7}{2}, \frac{13}{2} \right )\)

\(\left (\frac{-1}{4}, \frac{1}{2} \right )\)

\(\left (\frac{3}{2}, \frac{3}{2} \right )\)

1020183177

If \(x^2+y^2+2gx+2fy+1=0\) represents a pair of straight lines, then \(f^2+g^2\) is equal to


Please choose your answer from the right side options

\(0\)

\(1\)

\(2\)

\(4\)

\(3\)

1020183178

If \(\theta\) is the angle between the pair of straight lines \(x^2-5xy+4y^2+3x-4=0\) , then \(tan^2\,\theta\) is equal to


Please choose your answer from the right side options

\(\frac{9}{16}\)

\(\frac{16}{25}\)

\(\frac{9}{25}\)

\(\frac{21}{25}\)

\(\frac{25}{9}\)

1020183179

If \(3\hat{i}+2\hat{j}-5\hat{k}=x(2\hat{i}-\hat{j}+\hat{k})+y(\hat{i}+3\hat{j}-2\hat{k})+z(-2\hat{i}+\hat{j}-3\hat{k})\), then


Please choose your answer from the right side options

\(x=1,y=2,z=3\)

\(x=2,y=3,z=1\)

\(x=3,y=1,z=2\)

\(x=1,y=3,z=2\)

\(x=2,y=2,z=3\)

1020183180

\(sin\,15^{\circ}=\)


Please choose your answer from the right side options

\(\frac{\sqrt{3}-1}{2\sqrt{2}}\)

\(\frac{\sqrt{3}+1}{2\sqrt{2}}\)

\(\frac{1-\sqrt{3}}{2\sqrt{2}}\)

\(\frac{1+\sqrt{3}}{2\sqrt{2}}\)

\(\frac{-(1+\sqrt{3})}{2\sqrt{2}}\)

1020183181

If \(\bar{a}\) and  \(\bar{b}=3\hat{i}+6\hat{j}+6\hat{k}\) are collinear and \(\bar{a}\cdot \bar{b}=27\), then  \(\bar{a}\) is equal to


Please choose your answer from the right side options

\(3(\hat{i}+\hat{j}+\hat{k})\)

\(\hat{i}+2\hat{j}+2\hat{k}\)

\(2\hat{i}+2\hat{j}+2\hat{k}\)

\(\hat{i}+3\hat{j}+3\hat{k}\)

\(\hat{i}-3\hat{j}+2\hat{k}\)

1020183182

If  \(\left | \vec{a} \right |=13,\left | \vec{b} \right |=5 \,\,and\,\,\vec{a}\cdot \vec{b}=30\) , then \(\left | \vec{a}\times \vec{b} \right |\)is equal to


Please choose your answer from the right side options

\(30\)

\(\frac{30}{25}\sqrt{233}\)

\(\frac{30}{33}\sqrt{193}\)

\(\frac{65}{23}\sqrt{493}\)

\(\frac{65}{13}\sqrt{133}\)

1020183183

If \(^{56}P_{r+6}:\,^{54}P_{r+3}=30800:1\) , then \(r\) is equal to


Please choose your answer from the right side options

\(69\)

\(41\)

\(51\)

\(61\)

\(49\)

1020183184

Distance between two parallel lines \(y=2x+4\) and \(y=2x-1\) is


Please choose your answer from the right side options

\(5\)

\(5\sqrt5\)

\(\sqrt5\)

\(\frac{1}{5}\)

\(\frac{3}{\sqrt5}\)

1020183185

\((^7C_0\,+\,^7C_1)+(^7C_2\,+\,^7C_3)+...+(^7C_6\,+\,^7C_7)=\)


Please choose your answer from the right side options

\(2^8-2\)

\(2^7-1\)

\(2^7\)

\(2^8-1\)

\(2^7-2\)

1020183186

The coefficient of \(x\) in the expansion of \((1-3x+7x^2)(1-x)^{16}\) is


Please choose your answer from the right side options

\(17\)

\(19\)

\(-17\)

\(-19\)

\(20\)

1020183187

The equation of the circle with centre \((2,2)\) which passes through \((4,5)\) is


Please choose your answer from the right side options

\(x^2+y^2-4x+4y-77=0\)

\(x^2+y^2-4x-4y-5=0\)

\(x^2+y^2+2x+2y-59=0\)

\(x^2+y^2-2x-2y-23=0\)

\(x^2+y^2+4x-2y-26=0\)

1020183188

The point in the \(xy-\) plane which is equidistant from \((2,0,3),(0,3,2)\)and \((0,0,1)\) is


Please choose your answer from the right side options

\((1,2,3)\)

\((-3,2,0)\)

\((3,-2,0)\)

\((3,2,0)\)

\((3,2,1)\)

1020183189

Let \(f:x\rightarrow y\) be such that \(f(1)=2\) and \(f(x+y)=f(x)f(y)\) for all natural numbers \(x\) and \(y\) . If \(\sum_{k=1}^{n}f(a+k)=16(2^n-1)\) , then \(a\) is equal to


Please choose your answer from the right side options

\(3\)

\(4\)

\(5\)

\(6\)

\(7\)

1020183190

If \(^nC_{r-1}=36,\,^nC_{r}=84\) and \(^nC_{r+1}=126\) , then \(n=\)


Please choose your answer from the right side options

\(3\)

\(4\)

\(8\)

\(9\)

\(10\)

1020183191

Let  \(f:(-1,1)\rightarrow (-1,1)\)  be continuous, \(f(x)=f(x)^2\) for all \(x\,\epsilon \,(-1,1)\) and \(f(0)=\frac{1}{2}\) , then the value of  \(4f(\frac{1}{4})\) is


Please choose your answer from the right side options

\(1\)

\(2\)

\(3\)

\(4\)

\(5\)

1020183192

\(\lim_{x\rightarrow \infty }\sqrt{x^2+1}-\sqrt{x^2-1}=\)


Please choose your answer from the right side options

\(-1\)

\(1\)

\(0\)

\(2\)

\(4\)

1020183193

If \(f\) is differentiable at \(x=1\) and \(\lim_{h\rightarrow 0 }\frac{1}{h}f(1+h)=5,f'(1)=\)


Please choose your answer from the right side options

\(0\)

\(1\)

\(3\)

\(4\)

\(5\)

1020183194

The maximum value of the function \(2x^3-15x^2+36x+4\) is attained at


Please choose your answer from the right side options

\(0\)

\(3\)

\(4\)

\(2\)

\(5\)

1020183195

If \(\int f(x)\,cos\,xdx=\frac{1}{2}\left \{ f(x) \right \}^2+C\) , then \(f(\frac{\pi}{2})\) is


Please choose your answer from the right side options

\(C\)

\(\frac{\pi }{2}+C\)

\(C+1\)

\(2\pi+C\)

\(C+2\)

1020183196

\(\int_{\frac{\pi }{4}}^{\frac{3\pi }{4}}\frac{x}{1+sin\,x}dx=\)


Please choose your answer from the right side options

\(\pi(\sqrt2+2)\)

\(\pi(\sqrt2+1)\)

\(2\pi(\sqrt2-1)\)

\( 2\pi(\sqrt2+1)\)

\(\large \frac{\pi}{\sqrt2+1}\)

1020183197

\(\int_{0}^{\frac{\pi }{2}}\frac{2^{sin\,x}}{2^{sin\,x}+2^{cos\,x}}dx=\)


Please choose your answer from the right side options

\(2\)

\(\pi\)

\(\frac{\pi}{4}\)

\(2\pi\)

\(0\)

1020183198

\(\lim_{x\rightarrow 0}\left ( \frac{\int_{0}^{x^2}\,sin\,\sqrt{t}\,dt}{x^2} \right )=\)


Please choose your answer from the right side options

\(\frac{2}{3}\)

\(\frac{2}{9}\)

\(\frac{1}{3}\)

\(0\)

\(\frac{1}{6}\)

1020183199

The area bounded by \(y=sin^2x,x=\frac{\pi }{2} \) and \(x=\pi \) is


Please choose your answer from the right side options

\(\frac{\pi}{2}\)

\(\frac{\pi}{4}\)

\(\frac{\pi}{8}\)

\(\frac{\pi}{16}\)

\(2\pi\)

1220193110

If ω represents a complex cube root of unity, then \(\Large \overset{9}{\underset{r=1} \Sigma} r(r+1- \omega)(r +1 -\omega^2 ) =\)


Please choose your answer from the right side options

5025

4020

2016

3015

1220193111

If α and β are the roots of \(\large x^2 +7x +3 = 0\) and \(\Large \frac{2\alpha}{3-4\alpha} , \frac{2\beta}{3-4\beta}\) are the roots of \(\large ax^2 + bx +c=0\) and GCD of a,b,c is 1 then a +b + c =


Please choose your answer from the right side options

11

0

243

81

1220193112

If α, β are the roots of \(\large x^2 +bx+c=0\), γ , δ are the roots of \(\large x^2 + b_1 x + c_1 = 0\) and  \(\large \gamma < \alpha < \delta < \beta \) then \(\large (c-c_1)^2 <\)


Please choose your answer from the right side options

\(\large (b_1 -b)(bc_1-b_1c)\)

1

\(\large (b -b_1)^2\)

\(\large (c -c_1)(b_1c-b_1c_1)\)

1220193113

Let a,b,c be the sides of a scalene triangle. If λ is a real number such that root of the equation \(\large x^2 + 2(a+b+c)x + 3\lambda(ab+bc+ca)=0\) are real then the interval in which λ lies is


Please choose your answer from the right side options

\(\Large (- \infty, \frac{4}{3})\)

\(\Large ( \frac{5}{3}, \infty )\)

\(\Large (\frac{1}{3}, \frac{5}{3})\)

\(\Large (\frac{4}{3}, \infty)\)

1220193114

The polynomial equation of degree 4 having real coefficients with three of its roots as \(\large 2 \ \pm \sqrt 3\) and 1+ 2 i, is


Please choose your answer from the right side options

\(\large x^4 - 6x^3 - 14x^2 +22x +5=0\)

\(\large x^4 - 6x^3 - 19x+22x -5=0\)

\(\large x^4 - 6x^3 + 19x - 22x +5=0\)

\(\large x^4 - 6x^3 + 14x^2 -22x +5=0\)

1220193115

All letters of the word ANIMAL are permitted in all possible ways and the permutations thus formed are arranged in dictionary order. If the rank of word ANIMAL is x. Then the permutation with rank x , among the permutation obtained by permuting the letter of the word PERSON and arranging the permutations thus formed in the dictionary order is


Please choose your answer from the right side options

ENOPRS

NOSPRE

NOEPRS

ESORNP

1220193116

A student is allowed to choose atmost n books from a collection of 2n + 1 books. If the total number of ways in which he can select atleast one book is 255, then the value of n is


Please choose your answer from the right side options

4

5

6

7

1220193117

The sum of all the coefficients in the binomial expansion of \(\large (1+2x)^n\) is 6561. Let R = \(\large (1+2x)^n\) = I + F where I ∈ N and 0 < F < 1 . If \(\Large x= \frac{1}{\sqrt 2}\) , then \(\Large 1 - \frac{F}{1 + (\sqrt 2 - 1) ^4} =\)


Please choose your answer from the right side options

\(\large (3\sqrt 2 -4)\)

\(\large 4(3\sqrt 2 +4)\)

\(\large (\sqrt 2 -1)^4\)

1

1220193118

If \(\Large \frac{(1-px)^{-1}}{1-qx} = a_0 + a_1 x + a_2 x^2 + a_3 x^3 + ...,\) then \(\large a_n =\)


Please choose your answer from the right side options

\(\Large \frac{p^{n+1} -\ q^{n+1}}{q \ - \ p}\)

\(\Large \frac{p^{n+1} -\ q^{n+1}}{p \ - \ q}\)

\(\Large \frac{p^n -\ q^n}{q \ - \ p}\)

\(\Large \frac{p^n -\ q^n}{p \ - \ q}\)

1220193119

If \(\Large \frac{3}{(x-1)(x^2+x+1)} = \frac{1}{x-1} - \frac{x+2}{(x^2+x+1)} = f_1(x) - f_2(x)\) and \(\Large \frac{x+1}{(x-1)^2(x^2+x+1)} = Af_1(x) + ( B + \frac{D}{x-1}) f_2(x) + \frac{C}{(x-1)^2}\) then A + B + C + D =


Please choose your answer from the right side options

1

\(\Large - \frac{1}{3}\)

0

\(\Large \frac{1}{3}\)

1220193120

Let M and m respectively denote the maximum and minimum values of \(\large [f(\theta)]^2\) where \(\large f(\theta) = \sqrt{a^2 cos^2 \theta + b^2 sin^2 \theta } + \sqrt{a^2 sin^2 \theta + b^2 cos^2 \theta }\). Then M - m =


Please choose your answer from the right side options

\(\large a^2 + b^2\)

\(\large (a - b)^2\)

\(\large a^2 b^2\)

\(\large (a + b)^2\)

1220193121

If cos A = \(\Large - \frac{60}{61}\)and tan B =  \(\Large - \frac{7}{24}\) and neither A nor B is in the second quadrant, then the angle \(\Large A + \frac{B}{2}\) lies in the quadrant


Please choose your answer from the right side options

1

2

3

4

1220193122

\(\large cos^2\) 5° - \(\large cos^2\) 15° - \(\large sin^2 \) 15° + \(\large sin^2 \)35° + cos 15° sin 15° - cos 5° sin 35° =


Please choose your answer from the right side options

0

1

\(\Large \frac{3}{2}\)

2

1220193123

If cos θ ≠ 0 and sec θ -1 =( \(\large \sqrt 2\) - 1) tan θ then θ =


Please choose your answer from the right side options

\(\Large n \pi + \frac{\pi}{8}, n \ \epsilon \ Z\)

\(\Large 2n \pi + \frac{\pi}{4} \ (or)\ 2n\pi, n \ \epsilon \ Z\)

\(\Large 2n \pi + \frac{\pi}{8}, n \ \epsilon \ Z\)

\(\Large 2n \pi - \frac{\pi}{4} (or) \ 2n\pi, n \ \epsilon \ Z\)

1220193124

\(\large cot [\overset{32} {\underset{n=3} \Sigma} cot^{-1}(1 + \overset{n} {\underset{k=1} \Sigma} 2k)] =\)


Please choose your answer from the right side options

\(\Large \frac{10}{3}\)

\(\Large \frac{8}{3}\)

\(\Large \frac{14}{3}\)

\(\Large \frac{16}{3}\)

1220193125

If sin x cos hy = cosθ, cos x sin hy = sinθ and 4 tan x = 3. Then \(\large sin \ h^2y\) =


Please choose your answer from the right side options

\(\Large \frac{4}{5}\)

\(\Large \frac{9}{16}\)

\(\Large \frac{9}{25}\)

\(\Large \frac{16}{25}\)

1220193126

In triangle ABC, if \(\Large \frac{b+c}{9} = \frac{c+a}{10} = \frac{a+b}{11} = k \) then \(\Large \frac{cos \ A \ + \ cos \ B}{cos \ C} =\)


Please choose your answer from the right side options

\(\Large \frac{9}{10}\)

\(\Large \frac{10}{11}\)

\(\Large \frac{11}{12}\)

\(\Large \frac{12}{13}\)

1220193127

In triangle ABC, with usual notation, match the items in list I with the items in list II and choose the correct option

          List I                                  List II

A) \(\Large r_1 r_2 \sqrt{(\frac{4R -r_1 -r_2}{r_1 + r_2})}\)      I) b

B) \(\Large \frac{r_2 (r_3 + r_2)}{\sqrt{r_1r_2 + r_2r_3 + r_3r_1}}\)             II) \(\large a^2 , b^2 , c^2\) are in A.P

C) \(\Large \frac{a}{c} = \frac{sin \ (A-B)}{sin \ (B-C)}\)            III) \(\large \Delta\)

D) \(\large bc \ cos^2 \frac{A}{2}\)                          IV) \(\large Rr_1r_2r_3\)

                                                 V) s(s-a)

The correct match is


Please choose your answer from the right side options

A   B   C   D

IV  III   I    V

A   B   C   D

V   IV  III   II

A   B   C   D

III   I    II   V

A   B   C   D

IV  V    II   I

1220193128

If a ,b , c are the sides of ∆ABC for which \(\large r_1=8, \ r_2=12\) and \(\large r_3=24\)  then the ordered triad (a, b,c) =


Please choose your answer from the right side options

(12,20,16)

(12,16,20)

(16,12,20)

(20,16,12)

1220193129

If \(\large 4 \bar i+ 7 \bar j + 8 \bar k, \ 2 \bar i+ 3 \bar j + 4 \bar k, \ 2 \bar i+ 5 \bar j + 7 \bar k\) are respectively the positions vectors of vertices A,B,C of triangle ABC, then the position vectors of the point where the bisector of angle meets \(\large \overline {BC}\) is


Please choose your answer from the right side options

\(\Large 2 \bar i + \frac{13}{3} \bar j + 2 \bar k\)

\(\Large 2 \bar i - \frac{13}{3} \bar j + 6 \bar k\)

\(\Large 2 \bar i + 13 \bar j + 6 \bar k\)

\(\Large 2 \bar i + \frac{13}{3} \bar j + 6 \bar k\)

1220193130

The equation of the plane passing through the points \(\large \bar i + 2 \bar j - \bar k\) and perpendicular to the line of intersection of the plane \(\large \bar r . ( 3\bar i - \bar j + \bar k) = 1\) and   \(\large \bar r . ( \bar i + 4 \bar j -2 \bar k) = 2\)  is


Please choose your answer from the right side options

\(\large \bar r . ( -2\bar i - 5\bar j + \bar k) = 0\)

\(\large \bar r . ( \bar i + 7 \bar j + 4\bar k) = 0\)

\(\large \bar r . ( 2\bar i - 7\bar j -13 \bar k) = 1\)

\(\large \bar r . ( -2\bar i +7 \bar j + 13\bar k) = 0\)

1220193131

If the position vector of the vertices, A, B and C of \(\large \Delta ABC\) are \(\large \bar i + 2 \bar j - 5 \bar k, -2 \bar i + 2 \bar j + \bar k \) and  \(\large 2 \bar i + \bar j - \bar k\) respectively then ∠B =


Please choose your answer from the right side options

\(\Large cos^{-1} (\frac{7}{3\sqrt{10}})\)

\(\Large cos^{-1} (\frac{8}{\sqrt{105}})\)

\(\Large cos^{-1} (\frac{1}{\sqrt{42}})\)

\(\Large cos^{-1} (-\frac{7}{3\sqrt{10}})\)

1220193132

If the position vector of the vertices of \(\large \Delta ABC\) are \(\large \overline {OA} = 3 \bar i + \bar j + 2 \bar k ,\ \overline {OB} = \bar i + 2 \bar j + 3 \bar k\)  and \(\large \overline {OC} = 2 \bar i + 3 \bar j + \bar k\) then the length of altitude of triangle ABC drawn from A is


Please choose your answer from the right side options

\(\Large \sqrt{\frac{3}{2}}\)

\(\Large \frac{3}{\sqrt 2}\)

\(\Large \frac{\sqrt 3}{2}\)

\(\Large \frac{3}{2}\)

1220193133

A new tetrahedron is formed by joining the faces of a given tetrahedron OABC. Then the ratio of volume of new tetrahedron to that of given tetrahedron is


Please choose your answer from the right side options

\(\Large \frac{3}{25}\)

\(\Large \frac{1}{27}\)

\(\Large \frac{5}{62}\)

\(\Large \frac{1}{162}\)

1220193134

Let \(\large \bar A = 2 \bar i + \bar j - 2 \bar k\)  and  \(\large \bar B = \bar i + \bar j .\, if \, \bar C\)  is a vector such that \(\large \bar A\ . \overline C = |\overline C|, |\overline C - \bar A| = 2 \sqrt 2\)  and the angle between \(\large \bar A \times \overline B\) and \(\large \overline C\) is 30° then the value of  \(\large |(\bar A \times \overline B)\times \overline C|\)  is


Please choose your answer from the right side options

\(\Large \frac{2}{3}\)

\(\Large \frac{3}{2}\)

3

2

1220193135

If \(\Large a_0, a_1, ...a_{11}\) are in an arithmetic progression with common difference d , then their mean deviation from arithmetic mean is


Please choose your answer from the right side options

\(\Large \frac{30}{11} |d|\)

2|d|

3|d|

12 |d|

1220193136

The variance of the following continuous frequency distribution is 

Class Interval 0 - 10 10 - 20 20 - 30 30-40
Frequency  2 3 4 1

 


Please choose your answer from the right side options

201

62

19

84

1220193137

If two sections of strength 30 and 45 are formed from 75 students who are admitted in a school, then the probability that two particular students are always together in the same section is


Please choose your answer from the right side options

\(\Large \frac{66}{185}\)

\(\Large \frac{19}{37}\)

\(\Large \frac{29}{185}\)

\(\Large \frac{18}{37}\)

1220193138

A bag contains 2n coins out of which n-1 are unfair with head on both sides and remaining are fair. One coin is picked from a bag at random and tossed. If the probability that head falls in the toss is \(\Large \frac{41}{56}\) then the number of unfair coins in the bag is


Please choose your answer from the right side options

18

15

13

14

1220193139

Bag A contains 6 Green and 8 Red balls and bag B contains 9 Green and 5 Red balls. A card is drawn at random from a well shuffled pack of 52 playing cards. If it is a spade, two balls are drawn at random from bag A, otherwise two balls are drawn at random from bag B. If the two balls drawn are found to be of the same colour, then the probability that they are drawn from bag A is


Please choose your answer from the right side options

\(\Large \frac{43}{181}\)

\(\Large \frac{1}{4}\)

\(\Large \frac{48}{131}\)

\(\Large \frac{43}{138}\)

1220193140

A random variable X has the probability distribution,

X = \(\large x_i\)  1 2 3 4 5 6
P (X = \(\large x_i\) ) 0.2 0.3 0.12 0.1 0.2 0.08

If A = {\(\large x_i\) /\(\large x_i\)  is a prime number}, B ={ \(\large x_i\) /\(\large x_i\) < 4} are two events then P (A \(\large \cup\) B) =


Please choose your answer from the right side options

0.31

0.62

0.82

0.41

1220193141

In a Poisson distribution with mean \(\large \overset{\infty}{\underset{x=0} \Sigma} |x-\bar{x}| P(X=x) =\)


Please choose your answer from the right side options

e

\(\Large \frac{1}{e}\)

\(\Large \frac{2}{e}\)

\(\Large \frac{2}{3e}\)

1220193142

Two straight rods of length 2a and 2b move along the coordinate axis in such a way that their extremities are always concyclic. Then the locus of center of such circles is


Please choose your answer from the right side options

\(\large 2(x^2 + y^2) = a^2 + b^2\)

\(\large 2(x^2 - y^2) = a^2 + b^2\)

\(\large x^2 + y^2 = a^2 + b^2\)

\(\large x^2 - y^2 = a^2 - b^2\)

1220193143

When the coordinate axes are rotated around the origin in the positive direction through an
angle \(\Large \frac{\pi}{4}\), if the equation \(\large 25x^2 + 9 y^2 = 225\) is transformed to \(\large \alpha x^2 + \beta xy + \gamma y^2 = \delta\) the \(\large (\alpha + \beta + \gamma - \sqrt{\delta})^2 =\)


Please choose your answer from the right side options

3

9

4

16

1220193144

The equation of the line through the point of intersection of the lines 3x -4y + 1 = 0 and 5x + y -1 = 0 and making equal non-zero intercepts on the coordinate axes is


Please choose your answer from the right side options

2x + 2y = 3

23x + 23y = 6

23x + 23y = 11

2x + 2y = 7

1220193145

The line through P(a,2) where \(\large a \neq 0\), making an angle 45° with the positive direction of the X-axis meets the curve \(\Large \frac{x^2}{9} + \frac{y^2}{4} = 1\) at A and D and the coordinate axis at B and C. If PA, PB, PC and PD are in geometric progression then 2a =


Please choose your answer from the right side options

13

7

1

-13

1220193146

The equation of the perpendicular bisector of the sides AB and AC of a ΔABC are x - y = 5 and x+ 2y = 0 respectively. If A is (1, -2 ) then the equation of straight line BC is


Please choose your answer from the right side options

14x + 23y -40 = 0

12x + 17y - 28 = 0

14x - 29y -30 = 0

7x - 12y + 15 = 0

1220193147

If each line of a pair of lines original line passing through origin is at a perpendicular distance of 4 units from the point ( 3, 4 ), then the equation of the pair of lines is


Please choose your answer from the right side options

\(\large 7x^2 + 24xy =0\)

\(\large 7y^2 + 24xy =0\)

\(\large 7y^2 - 24xy =0\)

\(\large 7x^2 - 24xy =0\)

1220193148

Variable straight lines y = mx + c  make intercepts on the curve  \(\large y^2 - 4ax = 0\)  which subtend a right angle at the origin. Then the point of concurrence of these lines y = mx + c is


Please choose your answer from the right side options

(4a, 0)

(2a, 0)

(-4a, 0)

(-2a, 0)

1220193149

The abscissae of two points P, Q are the roots of the equation \(\large 2x^2 + 4x - 7= 0\) and their ordinates are the roots of the equation \(\large 3x^2 - 12x - 1= 0\) . Then the centre of the circle with PQ as a diameter is


Please choose your answer from the right side options

(-1, 2)

(-2, 6)

(1, -2)

(2, -6)

1220193150

If the angle between a pair of tangents drawn from a point P to the circle \(\large x^2 + y^2 + 4x -6y + 9sin^2 \alpha + 13cos^2 \alpha = 0\) is \(\large 2 \alpha\), then the equation of the locus of P is


Please choose your answer from the right side options

\(\large x^2 + y^2 + 4x -6y + 4 =0\)

\(\large x^2 + y^2 + 4x -6y - 9 =0\)

\(\large x^2 + y^2 - 4x + 6y - 4 =0\)

\(\large x^2 + y^2 + 4x -6y + 9 =0\)

1220193151

The equation of the circle whose radius is 3 and which touches internally the circle \(\large x^2 + y^2 - 4x - 6y -12 =0\) at the point (-1, -1 ) is


Please choose your answer from the right side options

\(\large 5x^2 + 5y^2 +9x -6y-7=0\)

\(\large 5x^2 + 5y^2 -8x -14y-32=0\)

\(\large 5x^2 + 5y^2 -6x +8y-8=0\)

\(\large 5x^2 + 5y^2 +6x -8y-12=0\)

1220193152

Suppose that the circle \(\large x^2 + y^2 +2gx +2fy +c =0\) has its centre on 2x + 3y - 7 =0 and cut the circles \(\large x^2 + y^2 -4x -6y +11 =0\) and \(\large x^2 + y^2 -10 x -4y + 21 =0\) orthogonally. Then 5g -10f +3c =


Please choose your answer from the right side options

0

1

3

9

1220193153

If the radical axis of the circle \(\large x^2 + y^2 + 2gx+ 2fy +c=0\) and \(\large 2x^2 + 2y^2 + 3x+ 8y +2c=0\) touches the circle \(\large x^2 + y^2 + 2x+ 2y +1=0\) then (4g - 3)(f - 2) =


Please choose your answer from the right side options

0

-1

1

2

1220193154

The parabola \(\large x^2 = 4ay\) makes an intercept of length \(\large \sqrt{40}\) units on the line y = 1 +2x then a value of 4a is


Please choose your answer from the right side options

2

-2

-1

2

1220193155

The locus of the point of intersection of perpendicular normals to the parabola \(\large y^2 =4 ax\) is


Please choose your answer from the right side options

\(\large y^2 -2ax + a^2=0\)

\(\large y^2 +ax + 2a^2=0\)

\(\large y^2 -ax + 2 a^2=0\)

\(\large y^2 -ax +3 a^2=0\)

1220193156

P is a variable point on the ellipse \(\Large \frac{x^2}{a^2} + \frac{y^2}{b^2}= 1\) with foci  \(\large F_1\) and  \(\large F_2\). If A is the area of the triangle \(\large PF_1F_2\), then the maximum value of A is


Please choose your answer from the right side options

\(\Large \frac{e}{ab}\)

\(\Large \frac{ae}{b}\)

\(\Large aeb\)

\(\Large \frac{ab}{e}\)

1220193157

If the line joining the points A (α) and B (β) on the ellipse \(\Large \frac{x^2}{25} + \frac{y^2}{9} = 1\) is a focal chord, then one possible value of \(\Large cot\frac{\alpha}{2} cot \frac{\beta}{2}\) is 


Please choose your answer from the right side options

-3

3

-9

9

1220193158

The equation of a tangent to the hyperbola \(\large 16x^2 - 25y^2 -96x + 100y - 356 = 0\) which makes an angle 45° with its transverse axis is


Please choose your answer from the right side options

x - y + 2 = 0

x - y + 4 = 0

x + y + 2 = 0

x + y + 4 = 0

1220193159

If P (0,7,10), Q(-1, 6, 6) and R(-4, 9, 6) are three points in the space, then PQR is


Please choose your answer from the right side options

Right angled isosceles triangle

Equilateral triangle

Isosceles but not right angled triangle

Scalene triangle

1220193160

A(2, 3, 5), B (\(\large \alpha\), 3, 3) and C(7, 5, \(\large \beta\)) are the vertices of a triangle. If the median through A is equally inclined with the coordinate axes then \(\Large Cos^{-1} (\frac{\alpha}{\beta})\) =


Please choose your answer from the right side options

\(\Large Cos^{-1} (\frac{-1}{9})\)

\(\Large \frac{\pi}{2}\)

\(\Large \frac{\pi}{3}\)

\(\Large Cos^{-1} (\frac{2}{5})\)

1220193161

The plane 3x+ 4y + 6z+ 7 = 0 is rotated about the line \(\large \bar r = (\bar i + 2 \bar j - 3 \bar k) + t(2\bar i - 3 \bar j + \bar k)\) until the plane passes through the origin. The equation of the plane in new position is


Please choose your answer from the right side options

x + y + z = 0

6x + 3y - 4z = 0

4x - 5y - 2z = 0

x + 2y + 4z = 0

1220193162

If \(\Large \underset{x \rightarrow \infty} {lim} \{ \frac{x^3 + 1}{x^2 + 1} - (\alpha x + \beta) \}\) exist and equal to 2 then the ordered pair (α , β) of real numbers is


Please choose your answer from the right side options

(1, -1)

(-2, 1)

(-1, 1)

(1, -2)

1220193163

For k>0, \(\Large \overset{\infty}{\underset{x=0} \Sigma} \frac{k^x}{x!} \underset{x \rightarrow \infty} {lim} \frac{n!}{(n-x)!} (1 - \frac{k}{n})^{n-x} (\frac{1}{n})^x = \)


Please choose your answer from the right side options

0

k

x

1

1220193164

Let \(\large f: \mathbb{R} \rightarrow \mathbb{R}\) be the function defined by

 \(\large f(x) = \left [ \begin{array}{ll} 5 & \mbox{if } x \leq 1 \\ a +bx & \mbox{if } 1< x < 3 \\ b +5x & \mbox{if } 3 \leq x <5 \\ 30 & \mbox{if } x \geq 5 \end{array} \right.\)  then f is

then  \(\large f\) is


Please choose your answer from the right side options

continuous if a = 5 and b = 5

continuous if a = 0 , b = 5

continuous if a = − 5, b =10

not continuous for any value of a and b

1220193165

Let [ x ] denote the greatest integer less than or equal to x. Then the number of points where the function y = [ x ] + |1 - x| , \(\large 1 \leq x \leq 3\) is not differentiable, is


Please choose your answer from the right side options

1

2

3

4

1220193166

If \(\large \sqrt{1 - x^6} + \sqrt{1-y^6} = a(x^3 - y^3)\) ,  then \(\Large y^2 \frac{dy}{dx} =\)


Please choose your answer from the right side options

\(\Large \sqrt{\frac{1-y^6}{1-x^6}}\)

\(\Large x \sqrt{\frac{1-y^6}{1-x^6}}\)

\(\Large x^2 \sqrt{\frac{1-y^6}{1-x^6}}\)

\(\Large \frac{1}{x^2} \sqrt{\frac{1-y^6}{1-x^6}}\)

1220193167

If y = f(x) is twice differentiable function such that a point P, \(\Large \frac{dy}{dx} =4, \frac{d^2y}{dx^2} = -3\), then \(\Large ( \frac{d^2x}{dy^2})_P = \)


Please choose your answer from the right side options

\(\Large \frac{64}{3}\)

\(\Large \frac{16}{3}\)

\(\Large \frac{3}{16}\)

\(\Large \frac{3}{64}\)

1220193168

The time T of oscillation of a simple pendulum of length L is governed by, \(\Large T =2\pi \sqrt{\frac{L}{g}}\)where g is constant. The percentage by which length be changed in order to correct an error of loss equal to 2 minutes of time per day is


Please choose your answer from the right side options

\(\Large - \frac{5}{18}\)

\(\Large - \frac{2}{9}\)

\(\Large \frac{1}{6}\)

\(\Large \frac{1}{9}\)

1220193169

Let A, G, H and S respectively denote the arithmetic mean, geometric mean, harmonic mean and the sum of the numbers \(\large a_1, a_2, a_3,...,a_n\). Then the value of x at which the function \(\large f(x) = \overset{n}{\underset{k=1} \Sigma} (x - a_k)^2\) has minimum is


Please choose your answer from the right side options

S

H

G

A

1220193170

For m > 1 , n > 1 , the value of c for which the Rolle’s theorem is applicable for the function \(\large f(x)=x^{2m-1} (a- x)^{2n}\) in (0, a) is


Please choose your answer from the right side options

\(\Large \frac{2am-1}{m+2n-1}\)

\(\Large \frac{a(m-n+1)}{2m+2n}\)

\(\Large \frac{a(2m-1)}{2m+2n-1}\)

\(\Large \frac{a(2m+1)}{m+n-1}\)

1220193171

If the function \(\large f : [-1,1] \rightarrow \mathbb{R}\) defined by

\(\large f(x) = \left\{ \begin{array}{ll} 2^x + 1 & \mbox{if } x \ \epsilon \ [-1,0) \\ 1 & \mbox{if } x = 0 \\ 2^x - 1 & \mbox{if } x \ \epsilon \ (0,1] \end{array} \right.\)then in [−1, 1] , f ( x ) has


Please choose your answer from the right side options

a maximum

a minimum

both maximum and minimum

neither maximum nor minimum

1220193172

\(\Large \int \frac{x-1}{(x+1)\sqrt{x^3 + x^2 +x}} dx=\)


Please choose your answer from the right side options

\(\Large 2 \ Tan^{-1} (\sqrt{\frac{1+x+x^2}{x}}) + c\)

\(\Large \ Tan^{-1} (\sqrt{\frac{1+x+x^2}{x}}) + c\)

\(\Large 2 \ Tan^{-1} (\sqrt{\frac{x}{1+x+x^2}}) + c\)

\(\Large \ Tan^{-1} (\sqrt{\frac{1+x^2}{x}}) + c\)

1220193173

If \(\large I(x)= \int x^2 (log \ x)^2 dx\) and I (1) = 0 , then I (x) =


Please choose your answer from the right side options

\(\Large \frac{x^3}{18}[8 (log \ x)^2 - 3 \ log \ x] + \frac{7}{18}\)

\(\Large \frac{x^3}{27}[9 (log \ x)^2 + 6 \ log \ x] - \frac{2}{27}\)

\(\Large \frac{x^3}{27}[9 (log \ x)^2 - 6 \ log \ x + 2] - \frac{2}{27}\)

\(\Large \frac{x^3}{27}[9 (log \ x)^2 - 6 \ log \ x - 2] + \frac{2}{27}\)

1220193174

\(\Large \int \frac{x^5 dx}{(x^2 + x +1)(x^6 + 1)(x^4 - x^3 + x -1)} =\)


Please choose your answer from the right side options

\(\Large log_e |\frac{x^6 - 1}{x^6 +1}|+ c\)

\(\Large \frac{1}{12}log_e |\frac{x^6 - 1}{x^6 +1}|+ c\)

\(\Large \frac{1}{12} log_e |\frac{x^4 + 1}{x^4 -1}|+ c\)

\(\Large log_e |\frac{x^8 + 4}{x^6 -1}|+ c\)

1220193175

\(\Large \int \frac{dx}{x + \sqrt{x-1}} =\)


Please choose your answer from the right side options

\(\Large log_e|x + \sqrt{x-1}| - \frac{1}{\sqrt 3}Tan^{-1} (\frac{2\sqrt{x-1} + 1}{\sqrt 3}) + c\)

\(\Large \frac{1}{\sqrt 3}log_e|x + \sqrt{x-1}| - Tan^{-1} (\frac{2\sqrt{x-1} + 1}{\sqrt 3}) + c\)

\(\Large \frac{2}{\sqrt 3}log_e|x + \sqrt{x-1}| - Tan^{-1} (\frac{2\sqrt{x-1} + 1}{\sqrt 3}) + c\)

\(\Large log_e|x + \sqrt{x-1}| - \frac{2}{\sqrt 3}Tan^{-1} (\frac{2\sqrt{x-1} + 1}{\sqrt 3}) + c\)

1220193176

\(\Large \overset{x}{\underset{log_e 2} \int}\frac{dt}{\sqrt{e^t - 1}} = \frac{\pi}{6} \implies x=\)


Please choose your answer from the right side options

\(\large 2 . log_e 2\)

\(\large 3 . log_e 2\)

\(\large 4 . log_e 2\)

\(\large 8 . log_e 2\)

1220193177

\(\Large \overset 1{\underset 0 \int} \frac{log_e (1+x)}{1+x^2} dx =\)


Please choose your answer from the right side options

\(\Large \frac{\pi}{4} log_e 2\)

\(\Large \frac{\pi}{6} log_e 6\)

\(\Large \frac{\pi}{2} log_e 8\)

\(\Large \frac{\pi}{8} log_e 2\)

1220193178

If the area of the circle \(\large x^2 + y^2 = 2\) is divided into parts by the parabola \(\large y = x^2\),  then the area (in sq. units) of the larger part is


Please choose your answer from the right side options

\(\Large \frac{3\pi}{2} - \frac{1}{3}\)

\(\Large 6\pi - \frac{4}{3}\)

\(\Large \frac{4\pi}{3} - \frac{2}{3}\)

\(\Large 4\pi - \frac{1}{4}\)

1220193179

If c is a parameter, then the differential equation of the family of curves \(\large x^2 = c(y+c)^2\) is


Please choose your answer from the right side options

\(\Large x(\frac{dy}{dx})^3 + y(\frac{dy}{dx})^2 - 1 = 0\)

\(\Large x(\frac{dy}{dx})^3 - y(\frac{dy}{dx})^2 + 1 = 0\)

\(\Large x(\frac{dy}{dx})^3 + y(\frac{dy}{dx})^2 + 1 = 0\)

\(\Large x(\frac{dy}{dx})^3 - y(\frac{dy}{dx})^2 - 1 = 0\)

1220193180

If \(\large f(x), f'(x), f''(x)\) are positive functions and \(\large f(0)=1, f'(0)=2\), then the solution of the differential equation  \(\large \left | {\begin{array}{cc} f(x) & f'(x) \\ f'(x) & f''(x) \\ \end{array} } \right | = 0 \) is


Please choose your answer from the right side options

\(\large e^{2x}\)

\(\large 2 \ sin \ x + 1\)

\(\large sin^2 x + 2x + 1\)

\(\large e^{4x}\)

1520173110

A bag contains 5 red balls, 3 black balls and 4 white balls are drawn at random. The probability that they are not of same colour is


Please choose your answer from the right side options

\(\frac{37}{44}\) 

\(\frac{31}{44}\) 

\(\frac{21}{44}\)

 \(\frac{41}{44}\)

1520173111

The radical centre of the circles

\(x^2+y^2-4x-6y+5=0,x^2+y^2-2x-4y-1=0,x^2+y^2-6x-2y=0 \) lies on the line 


Please choose your answer from the right side options

\(x+y-5=0 \) 

\(2x-4y+7=0 \)

\(4x-6y+5=0 \) 

\(18x-12y+1=0\)

1520173112

If \(cosec\theta -cot\theta =2017\) , then quadrant in which \(\theta\) lies is 


Please choose your answer from the right side options

IV 

III 

II

1520173113

If \(\int e^{2x}f'(x)dx=g(x) \) ,  then \(\int (e^{2x}f(x)+e^{2x}f'(x))dx= \)


Please choose your answer from the right side options

\(\frac{1}{2}[e^{2x}f(x)-g(x)]+C \)

\(\frac{1}{2}[e^{2x}f(x)+g(x)]+C \)

\(\frac{1}{2}[e^{2x}f(2x)+g(x)]+C \)

'>

\(\frac{1}{2}[e^{2x}f'(2x)+g(x)]+C\)

1520173114

If \(A=(5,3),B=(3,-2)\) and a point P is such that the area of the triangle PAB is 9, then the locus of P represents 


Please choose your answer from the right side options

a circle 

a pair of coincident lines 

a pair of parallel lines 

a pair of perpendicular lines